MBE QUESTIONS + ANSWER Flashcards

1
Q

The landowner is privileged to protect her property from intrusion by a means not intended or likely to cause death or serious bodily harm. The fact that the barbed wire presents its own warning and is not a hidden trap makes it a reasonable device for discouraging trespassers.

A

A landowner who owned a large tract of land in the mountains sought to protect a herd of wild deer that lived on part of the land. Although the landowner had posted signs that said, “No Hunting-No Trespassing,” hunters frequently intruded to kill the deer. Recently, the landowner built an eight-foot-high chain-link fence, topped by three strands of barbed wire, across a gully on her land that provided the only access to the area where the deer lived. A wildlife photographer asked the landowner for permission to enter the land to photograph the deer. Because the landowner feared that any publicity would encourage further intrusions by hunters, she denied the photographer’s request. Frustrated, the photographer attempted to climb the fence. He became entangled in the barbed wire and suffered extensive lacerations. The wounds became infected and ultimately caused his death. The photographer’s personal representative has sued the landowner. Is the personal representative likely to prevail? No, because the potential for harm created by the presence of the barbed wire was apparent.

How well did you know this?
1
Not at all
2
3
4
5
Perfectly
2
Q

Once a trial judge has definitively ruled on an evidentiary motion in limine, either ruling to admit or to deny evidence, the issue of the evidence’s admissibility is preserved for appeal and need not be raised again at trial. See Fed. R. Evid. 103(a), advisory committee’s note. However, when a trial judge makes a preliminary evidentiary ruling that is contingent on some showing being made at trial, the failure to object at trial effectively waives the issue on appeal. See Wilson v. Williams, 182 F.3d 562, 565-66 (7th Cir. 1999).

A

A plaintiff brought a diversity action in federal court, seeking to recover damages for an injury he sustained in a car accident with the defendant. In a pretrial motion, the defendant’s attorney argued that the trial judge should exclude evidence of an incriminating statement made by the defendant at the scene of the accident. The plaintiff objected to the exclusion of this evidence, and the parties fully briefed the issue. The trial judge considered the parties’ briefs and issued a preliminary ruling denying the defendant’s motion to exclude and allowing the evidence to be admitted. The judge’s ruling cautioned the plaintiff that, before introducing the statement at trial, the plaintiff would have to first introduce certain other foundational evidence as a condition to the statement’s admissibility. At trial, the plaintiff first introduced the foundational evidence and then introduced the defendant’s incriminating statement, without which there turned out to be little evidence of the defendant’s liability. The defendant’s attorney did not object to the admission of the statement or otherwise reassert the defendant’s pretrial evidentiary argument. The defendant was found liable to the plaintiff and then appealed the judgment based on the pretrial ruling allowing the statement to be admitted. Is the appellate court likely to hear the appeal? No, because the defendant’s attorney did not object to the admission of the incriminating statement at trial in order to preserve the issue for appeal.

How well did you know this?
1
Not at all
2
3
4
5
Perfectly
3
Q

Whether a battery defendant’s conduct was reasonable under the circumstances is irrelevant if in fact the defendant intended to make a harmful or offensive contact with the plaintiff. It would be relevant in a negligence action, but not in a battery action.

A

An ordinance in a small town required all restaurants to designate smoking and nonsmoking sections for their customers. A cigarette smoker and a nonsmoker were seated at adjoining tables in a small restaurant. The smoker’s table was in the smoking section, and the nonsmoker’s table was in the nonsmoking section. When the smoker lit a cigarette, the nonsmoker politely requested that he not smoke, explaining that she had a severe allergy to cigarette smoke. The smoker ignored the nonsmoker’s request and continued to smoke. As a result, the nonsmoker was hospitalized with a severe allergic reaction to the smoke. The nonsmoker brought a battery action against the smoker. Which of the following questions will NOT be an issue in the battery action? Was the smoker’s conduct unreasonable under the circumstances?

How well did you know this?
1
Not at all
2
3
4
5
Perfectly
4
Q

A performance that is subject to an express condition cannot become due unless the condition occurs or its nonoccurrence is excused. The detective’s entitlement to the reward was subject to two conditions-the arrest and the conviction of the fugitive. The first condition was satisfied when the detective delivered the fugitive to the authorities. The second condition did not occur, but its nonoccurrence is excused under the doctrine of prevention, which requires that a party refrain from conduct that prevents or hinders the occurrence of a condition. In this case, the authorities themselves prevented the conviction from occurring.

A

A fugitive was wanted for murder. The authorities offered the following reward: “$20,000 to anyone who provides information leading to the arrest and conviction of this fugitive.” A private detective knew of the reward, located the fugitive, and brought him to the authorities, who arrested him. The authorities then determined that while the fugitive had, in fact, committed the crime, he had been directed to commit the crime by his boss. The authorities and the fugitive then agreed that in exchange for the fugitive’s testimony against his boss, all charges against the fugitive would be dropped. The fugitive testified and was released. The authorities refused to pay the reward to the private detective on the ground that the fugitive was never convicted. Would the private detective be likely to prevail in a breach of contract action against the authorities? Yes, because the authorities themselves prevented the conviction of the fugitive.

How well did you know this?
1
Not at all
2
3
4
5
Perfectly
5
Q

A confession made by a person under arrest and subjected to interrogation can be admitted into evidence at trial only if Miranda warnings were properly given beforehand. Because the defendant contends that no Miranda warnings were given, she is entitled to a hearing on the issue. Under Rule 104(c)(1) of the Federal Rules of Evidence, the hearing must be conducted so that the jury cannot hear it.

A

A defendant was on trial for burglary. The prosecutor called the arresting officer to testify that shortly after her arrest and interrogation, the defendant had orally admitted her guilt to the officer. Before the officer testified, the defendant objected that no Miranda warnings had been given to her, and she requested a hearing outside the presence of the jury to hear evidence on that issue. How should the court proceed? The court should grant the request, because the hearing on the admissibility of the confession must be conducted outside the presence of the jury.

How well did you know this?
1
Not at all
2
3
4
5
Perfectly
6
Q

Four elements are required to bar subsequent claims under the doctrine of res judicata: (1) there must be a final judgment on the merits, (2) rendered by a court of competent jurisdiction, (3) involving the same parties (or parties in privity), and (4) involving the same claims (which includes the same transaction, incident, or nucleus of operative facts). Here, the nucleus of operative facts was identical in both actions, i.e., the car accident that injured the plaintiff gave rise to both claims. The subsequent change in the plaintiff’s prognosis does not alter the original facts related to the car accident, which facts would have to be relitigated in the negligence action to establish the manufacturer’s liability.

A

A plaintiff brought a products liability action against an automotive parts manufacturer, alleging that the manufacturer produced a defective automotive part that caused an automobile accident. The plaintiff sought damages relating to injuries that she sustained in the accident. The plaintiff’s injuries included a head injury from which the plaintiff’s doctor, who testified at trial, believed she would ultimately make a full recovery. The action resulted in a final judgment and damages award for the plaintiff. Three years later, the plaintiff was still experiencing headaches and blurred vision from the head injury that she sustained in the accident. She went to a new doctor, an expert in head injuries, who believed that she would experience these symptoms for the rest of her life. If the plaintiff brings a negligence action against the defendant manufacturer and seeks additional damages because of her chronic condition, is the subsequent action likely to be barred by the doctrine of res judicata? Yes, because the nucleus of operative facts giving rise to the plaintiff’s chronic condition is identical to the facts that gave rise to the previous action.

How well did you know this?
1
Not at all
2
3
4
5
Perfectly
7
Q

The space on city buses used for the posting of placards qualifies as a designated public forum because it is public property that the city has decided to open for an expressive use. The organization’s placard was consistent with the city’s designated use of the forum. The city administrator’s denial of space to the organization was based on the content of the placard and therefore triggered strict scrutiny, which requires that the denial be necessary to serve a compelling government interest. The reasons cited for the city’s denial of the organization’s request do not implicate compelling government interests that would justify a content-based speech restriction.

A

A city owned and operated a municipal bus system. The city sold space on its buses for the posting of placards. Under the relevant city ordinance, the administrator of the bus system had sole discretion to decide which placards could be posted on the buses, and the administrator’s decision was final. Although most of the placards that appeared on city buses were commercial advertisements, the administrator had often sold space on the buses for placards promoting various political, charitable, and religious causes. After a circus bought space on the buses for placards advertising its upcoming performances, an animal rights organization asked to buy space for a placard with photographs showing the mistreatment of animals in circus shows. The administrator denied the organization’s request, stating that the proposed placard would be offensive to the circus, which had paid a substantial sum to place its placards on the buses, and that a circus employee had told her that none of the photographs on the organization’s placard depicted animals belonging to this particular circus. The organization sued the administrator in an appropriate court for a declaration that her denial of the organization’s request for placard space for the reasons she gave violated the First Amendment as made applicable to the states by the Fourteenth Amendment. Is the organization likely to prevail? Yes, because a public official may not refuse to permit the dissemination of a message in a public forum solely on the basis of its content unless that denial is necessary to serve a compelling government interest.

How well did you know this?
1
Not at all
2
3
4
5
Perfectly
8
Q

There has been no finding of negligence on the part of the government. The trier of fact found that the government had selected a reliable manufacturer for the component part and could not have anticipated or prevented the malfunction. Res ipsa loquitur applies only to situations in which a lay jury could say that the accident would not ordinarily occur in the absence of the defendant’s negligence. This is not such a situation because a nuclear reactor is complex machinery beyond the ordinary expertise of a lay jury. Also, another potential defendant (the engineering company) is involved, and the findings are inconsistent with a conclusion that this is the sort of accident that would not ordinarily occur in the absence of the government’s negligence.

A

Under the Federal Tort Claims Act, with certain exceptions not relevant here, the federal government is liable only for negligence. A federally owned and operated nuclear reactor emitted substantial quantities of radioactive matter that settled on a nearby dairy farm, killing the dairy herd and contaminating the soil. At the trial of an action brought against the federal government by the farm’s owner, the trier of fact found the following: (1) the nuclear plant had a sound design, but a valve made by an engineering company had malfunctioned and allowed the radioactive matter to escape; (2) the engineering company was universally regarded as a quality manufacturer of components for nuclear plants; and (3) there was no way the federal government could have anticipated or prevented the emission of the radioactive matter. If there is no other applicable statute, for which party should the court enter judgment? The government, on the ground that a case under the Federal Tort Claims Act has not been proved.

How well did you know this?
1
Not at all
2
3
4
5
Perfectly
9
Q

The law regulates only commercial speech, and the First Amendment invalidates any law regulating such speech unless the law is narrowly tailored to serve a substantial government interest. The U.S. Supreme Court has held that a law barring the solicitation of accident victims within a limited time period following an accident was narrowly tailored to serve the state’s substantial interest in protecting the privacy of the victims. The law at issue is not a time, place, and manner regulation because it restricts speech based on its content. Because the law is a content-based regulation of commercial speech, it is valid only if it is narrowly tailored to serve a substantial government interest.

A

A state legislature received complaints from traffic accident victims who, in the days immediately following their accidents, had received unwelcome and occasionally misleading telephone calls on behalf of medical care providers. The callers warned of the risks of not obtaining prompt medical evaluation to detect injuries resulting from accidents and offered free examinations to determine whether the victims had suffered any compensable injuries. In response to these complaints, the legislature enacted a law prohibiting medical care providers from soliciting any accident victim by telephone within 30 days of his or her accident. Which of the following arguments would be most helpful to the state in defending the constitutionality of the law? The state has substantial interests in protecting the privacy of accident victims and in regulating the practice of medical care providers, and the law is narrowly tailored to achieve the state’s objectives.

How well did you know this?
1
Not at all
2
3
4
5
Perfectly
10
Q

Under the Model Penal Code, a negligent homicide occurs because of the defendant’s disregard of a risk of which the defendant should have been aware. Here, the driver should have been aware of the danger of taking her eyes off the road. The remaining answer options are incorrect because the facts do not support a finding of intent, recklessness, or gross negligence. Involuntary manslaughter requires either a reckless or grossly negligent act, or the commission of a misdemeanor. Here, there is no indication that the defendant consciously disregarded a known risk (and thus acted recklessly) or that the defendant did anything illegal. Instead, the most serious supportable homicide charge is negligent homicide under the Model Penal Code. Under the Model Penal Code, a negligent homicide occurs because of the defendant’s disregard of a risk of which the defendant should have been aware. Here, the driver should have been aware of the danger of taking her eyes off the road.

A

A driver accidentally spilled some coffee in her lap while driving on a low-congestion suburban street. While glancing down at the spill, the driver failed to notice a pedestrian step into the road. The driver hit and killed the pedestrian. Which of the following is the most serious supportable homicide charge against the driver? Negligent homicide under the Model Penal Code

How well did you know this?
1
Not at all
2
3
4
5
Perfectly
11
Q

Under Rule 411 of the Federal Rules of Evidence it is true that evidence that a person was insured is generally not admissible to prove that the person acted negligently or otherwise wrongfully. However, such evidence may be received for other purposes, and proof of motive is such a purpose. In this respect, Rule 411 is similar to Rule 404(b), under which evidence of past conduct cannot be admitted to prove a propensity to engage in such conduct but can be admitted on the issue of motive.

A

A defendant’s house was destroyed by fire, and she was charged with arson. To prove that the defendant had a motive to burn down her house, the government offered evidence that the defendant had fully insured the house and its contents. Should the court admit this evidence? Yes, because evidence of insurance on the house has a tendency to show that the defendant had a motive to burn down the house.

How well did you know this?
1
Not at all
2
3
4
5
Perfectly
12
Q

UCC § 2-613 provides that where goods identified at the time the contract was made are totally destroyed before the risk of their loss has passed to the buyer and without the fault of either party, the contract is avoided and each party is relieved of its respective obligation to perform. Under UCC § 2-501, the goods were identified at the time of contract formation because the parties agreed to the delivery of a specific automobile. In addition, the car was destroyed without the fault of either party and before the risk of loss had passed. Therefore the contract is avoided. Because each party’s performance is discharged, neither party can assert a valid claim against the other. UCC § 2-613 provides that where goods identified at the time the contract was made are totally destroyed before the risk of their loss has passed to the buyer and without the fault of either party, the contract is avoided and each party is relieved of its respective obligation to perform. The avoidance of the contract excused both parties’ performance obligations. Consequently neither party has a claim against the other.

A

On June 1, a seller agreed to sell an antique car to a buyer for $20,000, in a writing signed by both the seller and the buyer. At the time, the car was on display in a museum in a different city and was to be delivered to the buyer on August 1. On July 15, before the risk of loss had passed to the buyer, the car was destroyed by fire without fault of either party. Subsequent to the contract but before the fire, the car had increased in value to $30,000. The seller sued the buyer for the contract price of $20,000, and the buyer counterclaimed for $30,000. What is the likely outcome of this suit? Both claims will fail.

How well did you know this?
1
Not at all
2
3
4
5
Perfectly
13
Q

A bill of attainder is a legislative act that singles out particular individuals for punishment without a trial; bills of attainder are explicitly prohibited by the Constitution. In United States v. Lovett, 328 U.S. 303 (1946), the U.S. Supreme Court held that a statute barring particular individuals from government employment qualified as punishment within the meaning of the constitutional provision prohibiting bills of attainder. Although due process entitles an individual to notice and a hearing before being deprived of an interest in liberty or property, these requirements do not apply to legislative acts. It is the bill of attainder clause that imposes these requirements on Congress. In United States v. Lovett, 328 U.S. 303 (1946), the U.S. Supreme Court held that a statute barring particular individuals from government employment qualified as punishment within the meaning of the constitutional provision prohibiting bills of attainder.

A

A Senate investigative committee released a report identifying three U.S. citizens as individuals who were organizing support for terrorist activities. All three were employed by the U.S. government as park rangers. In response, Congress enacted a statute naming these three individuals and providing that they could not hold any position of employment with the federal government. Which of the following constitutional provisions provides the best means for challenging the constitutionality of the statute? The bill of attainder clause.

How well did you know this?
1
Not at all
2
3
4
5
Perfectly
14
Q

The warrant was valid, but its validity was triggered by and limited to the delivered package. Accordingly, once the only object of that search was discovered, the warrant did not authorize a further exploratory search of the house. The search warrant was valid, but its validity was triggered by and limited to the delivered package. Accordingly, once the only object of that search—the package—was discovered, the warrant did not authorize a further exploratory search of the house.

A

United States customs officials received an anonymous tip that heroin would be found inside a distinctively marked red package mailed from a foreign country to a particular address in the United States. Pursuant to this tip, United States customs officers intercepted and opened the red package and found heroin inside. They then resealed the package and left the heroin inside it. The FBI was notified and, as agents watched, the package was delivered to the address. The FBI then secured a warrant to search the house for the package. About two hours after the package was delivered, agents executed the warrant at the house. The man who opened the door was arrested, and the agents found the package, unopened, in an upstairs bedroom closet. After seizing the package, the agents looked through the rest of the house. They found a machine gun in a footlocker in the basement. The man was charged with unlawful possession of the machine gun, among other crimes. He moved to suppress the use of the gun as evidence. Should the court grant the motion to suppress the machine gun?

How well did you know this?
1
Not at all
2
3
4
5
Perfectly
15
Q

A deceased person cannot take and hold title to property. If a named beneficiary predeceases the testator and there is no provision in the will for what happens to the gift in that case, the gift to that beneficiary lapses. In this case, the gift to the friend lapsed. The gift of the residence was a specific gift, and the lapse of this specific gift passes the residence through the residuary clause of the will. The charity is the residuary taker. There is no applicable anti-lapse statute which might have substituted the friend’s child as the beneficiary of the bequest if the friend were a protected beneficiary under the statute.

A

A man died testate. The man’s estate consisted of a residence as well as significant personal property. By his duly probated will, the man devised the residence to a friend, who was specifically identified in the will. The residue of the estate was given to a stated charity. The man’s friend, although alive at the time the man executed the will, had predeceased the man. The friend’s wife and their child, who has a disability, both survived the man. The value of the residence has increased significantly because of recent zoning changes. There is credible extrinsic evidence that the man wanted his friend to own the residence after the man’s death so that the friend and his wife could care for their child there. There is no applicable statute. If both the charity and the child claim the residence, to whom should the estate distribute the residence? The charity, because the devise to the friend lapsed.

How well did you know this?
1
Not at all
2
3
4
5
Perfectly
16
Q

The court will exercise strict scrutiny only if the challenger can show that the government action targeted the religious practice in question. A court typically invalidates government action at strict scrutiny. A simple benefit-burden balance of the government action at issue does not establish a violation of the free exercise clause. In order to establish a free exercise violation, the challenger must show that the government action targeted the religious practice in question.

A

Several sites on a mountain within federal public lands are regarded as sacred to a group of people that has gathered there for years to perform religious ceremonies. The United States Forest Service recently issued a permit to a private developer to construct a ski facility in an area that includes the sites that are sacred to the group. The group has filed suit in federal district court against the Forest Service to force cancellation of the permit, claiming solely that the permit violates its First Amendment right to the free exercise of religion. The Forest Service has conceded that the group’s religious beliefs are sincere and that construction of the ski facility will adversely affect the group’s religious practices. What must the group show to prevail on its First Amendment claim? The permit issued by the government is aimed at suppressing the religious practices of the group.

How well did you know this?
1
Not at all
2
3
4
5
Perfectly
17
Q

Rule 15 of the Federal Rules of Civil Procedure (FRCP) provides that a party may amend its pleading “once as a matter of course” before trial within either: (1) 21 days after serving it, or (2) if the pleading sought to be amended is one that requires the opposing party’s response, then 21 days after the responsive pleading is served or 21 days after service of a motion under Rule 12(b), (e), or (f), whichever is earlier. Generally, a motion that attacks and challenges a pleading is not a “responsive pleading” as defined under Rule 7. However, Rule 15 names three types of pre-answer motions that are treated like responsive pleadings, which will mark the beginning of the 21-day period during which the opposing party has the right to amend: (1) a motion to dismiss under Rule 12(b); (2) a motion for a more definite statement under Rule 12(e); and (3) a motion to strike under Rule 12(f). SeeFRCP 15(a)(1)(B). Here, the plaintiff filed a motion to dismiss for lack of subject-matter jurisdiction under Rule 12(b)(1). The plaintiff’s motion is one of the three types of motions under Rule 15(a)(1)(B) that start the clock on the defendant’s 21-day period to amend as a matter of course. See FRCP 15(a)(1)(B); see alsoFRCP 15, advisory committee’s notes (2009). Because the defendant filed an amendment within 21 days of the plaintiff’s motion to dismiss, the amendment is proper.

A

A plaintiff sued a defendant for trademark infringement in federal court. The defendant answered the complaint, denying all allegations and asserting a counterclaim against the plaintiff. Four days later, the plaintiff moved to dismiss the counterclaim for lack of subject-matter jurisdiction pursuant to Rule 12(b)(1) of the Federal Rules of Civil Procedure. Twenty days after the plaintiff filed his motion to dismiss and while the motion was still pending, the defendant amended his counterclaim without leave of court. Is the defendant’s amendment proper under the Federal Rules of Civil Procedure? Yes, because it was filed within 21 days of the plaintiff’s motion to dismiss.

How well did you know this?
1
Not at all
2
3
4
5
Perfectly
18
Q

When changing the name of a party, Rule 15(c)(1)(C) of the Federal Rules of Civil Procedure provides that an amendment relates back to the date of the original filing if both of the following occur within the period allowed for serving the summons: (1) the proposed new defending party must receive sufficient notice of the lawsuit, so that it will not be materially hindered in defending itself on the merits if the amendment is allowed to relate back; and (2) the proposed new defending party must have known, or have had reason to know, that the plaintiff made a mistake as to the identity of the true defendant, and that it would have been sued initially if not for the mistake.

A

A shopper tripped and fell over a broken curb in a shopping center parking lot. A large sign in the parking lot identified a limited liability company named Avenue, LLC (the LLC) that operated and maintained the lot. A large construction corporation named Avenue Corporation (the corporation) wholly owned the LLC and had the same business address, principals, managers, and lawyers. The shopper mistakenly sued the corporation just one day before the statute of limitations on her claim expired. The shopper’s attorney had seen the sign in the parking lot, but made a mistake when listing the name of the defendant in the shopper’s complaint. The shopper’s attorney served the summons and complaint on both the corporation and the LLC. The LLC realized immediately that the shopper had made a mistake. A week later, the shopper amended her complaint. The amended complaint changed the name of the defendant to Avenue, LLC. The applicable statute that defines the statute of limitations would allow the amendment. Nevertheless, the LLC opposed the amendment and argued that the claims against it should not relate back to the original filing date. What is the most likely outcome? The court will allow the amendment to relate back to the original filing date, because the LLC received notice of the lawsuit, would not be hindered in defending itself, and knew or had reason to know that it would have been sued initially, but for the mistake in identifying the corporation as the defendant.

How well did you know this?
1
Not at all
2
3
4
5
Perfectly
19
Q

UCC § 2-206(1)(b) provides that a seller’s shipment of nonconforming goods with a notice of accommodation does not constitute an acceptance and breach, but rather a counteroffer, which the buyer is free to either accept or reject. Section 2-206(1)(b) also provides, however, that a contract calling for prompt shipment can be accepted either by a prompt promise to ship or by the prompt shipment of goods. The seller accepted the buyer’s offer by a promise to ship when he mailed his June 2 letter. UCC § 2-601 allows a buyer to accept or reject nonconforming goods and, in either event, to recover damages. The buyer has an action for breach because the computer shipped on June 3 failed to conform to the contract formed on June 2 when the seller mailed his letter of acceptance. The mailing of the notice of accommodation is irrelevant because the seller accepted the buyer’s offer by promising, in his June 2 letter, to ship the computer. The buyer can accept or reject the nonconforming computer and can recover damages, if any, for breach.

A

On June 1, a seller received a mail order from a buyer requesting prompt shipment of a specified computer model at the seller’s current catalog price. On June 2, the seller mailed to the buyer a letter accepting the order and assuring the buyer that the computer would be shipped on June 3. On June 3, the seller realized that he was out of that computer model, shipped a different computer model to the buyer, and mailed a separate notice of accommodation. On June 5, the buyer received the seller’s June 2 letter and the different computer model, but not the notice of accommodation. On June 5, which of the following is a correct statement of the parties’ legal rights and duties? The buyer can either accept or reject the different computer model and, in either event, recover damages, if any, for breach of contract.

How well did you know this?
1
Not at all
2
3
4
5
Perfectly
20
Q

A federal jury must begin with six to 12 members, selected from a fair cross-section of the community. A verdict must be returned by a jury of at least six members. The verdict must be unanimous unless the parties stipulate otherwise. Here, the verdict must be unanimous because the defendant has not agreed otherwise. Note that it is the unanimity of the jurors causing an issue in this case, not the total number of jurors approving the verdict. If only six jurors had been impaneled, it would be proper for the jury to return a unanimous verdict by those six jurors. A federal jury must begin with six to 12 members, selected from a fair cross-section of the community. A verdict must be returned by a jury of at least six members. The verdict must be unanimous unless the parties stipulate otherwise. Here, the verdict must be unanimous because the defendant has not agreed otherwise.

A

A plaintiff brought an employment discrimination action in federal court against her former employer, alleging that the defendant had terminated the plaintiff on the basis of her age. The plaintiff sought $500,000 in damages. Following voir dire, and with no objections from either party, the judge impaneled a jury of eight people. During trial, at the close of the evidence, the plaintiff requested that the judge instruct the jury that it could return a verdict for the plaintiff if six or more of the eight jurors found that the defendant was liable. Would such an instruction be proper over the defendant’s objection? No, because a federal jury must return a unanimous verdict unless the parties stipulate otherwise.

How well did you know this?
1
Not at all
2
3
4
5
Perfectly
21
Q

The usual rule prohibiting Congress from enacting a statute overruling a constitutional decision of the U.S. Supreme Court does not apply to enactments based on Congress’s commerce power because the Constitution gives Congress plenary authority to regulate conduct that is within the commerce power. The statute permitting any state to regulate the degree of light reflectiveness of the exteriors of commercial trucks using the state’s highways is a valid enactment of the commerce power because commercial trucks are instrumentalities of interstate commerce.

A

A state legislature conducted an investigation into a series of fatal accidents in the state involving commercial trucks with exteriors made of polished aluminum. The investigation revealed that the sun’s glare reflecting off these trucks blinded the drivers of other vehicles. In response, the state’s legislature enacted a law prohibiting commercial trucks with polished aluminum exteriors from traveling on the state’s highways. Litigation over the state law resulted in a final decision by the United States Supreme Court that the law impermissibly burdened interstate commerce and therefore was unconstitutional. Congress later enacted a statute permitting any state to enact a law regulating the degree of light reflectiveness of the exteriors of commercial trucks using the state’s highways. Is this federal statute constitutional? Yes, because Article I, Section 8 grants Congress authority to enact statutes authorizing states to impose burdens on interstate commerce that would otherwise be prohibited.

How well did you know this?
1
Not at all
2
3
4
5
Perfectly
22
Q

The law supports the settlement of debts and claims. However, consideration is required for a settlement to be enforceable. Under the preexisting duty rule, the creditor’s promise to forbear from suing to collect was not supported by consideration from the debtor, because the amount due was liquidated and the debtor promised to do nothing more than he was already obligated to do. The creditor’s promise was not supported by consideration from the debtor because it allowed for payment of an undisputed amount, $1,000, after the time for payment of the debt had passed.

A

A debtor’s liquidated and undisputed $1,000 debt to a creditor was due on March 1. When the debt was still unpaid on March 15, the creditor told the debtor that if the debtor promised to pay the $1,000 on or before December 1, then the creditor would not sue to collect the debt. The debtor orally agreed. On April 1, the creditor sued the debtor to collect the debt that had become due on March 1. The debtor moved to dismiss the creditor’s complaint. Should the court grant the debtor’s motion? No, because there was no consideration to support the creditor’s promise not to sue.

How well did you know this?
1
Not at all
2
3
4
5
Perfectly
23
Q

Whether an invasion constitutes a nuisance turns on whether it causes significant harm of a kind that would be suffered by a normal member of the community. Here, only one abnormally sensitive person was disturbed by the chimes.

A

A recently established law school constructed its building in a quiet residential neighborhood. The law school had obtained all the necessary municipal permits for the construction of the building, which included a large clock tower whose clock chimed every hour. The chimes disturbed only one homeowner in the neighborhood, who had purchased her house prior to the construction of the building. The homeowner was abnormally sensitive to ringing sounds, such as bells and sirens, and found the chimes to be extremely annoying. In a nuisance action by the homeowner against the law school, will the homeowner be likely to prevail? No, because the chimes do not disturb the other residents of the neighborhood.

How well did you know this?
1
Not at all
2
3
4
5
Perfectly
24
Q

The woman cannot be found guilty of murder, because the hit man did not cause the neighbor’s death, but she can be convicted of attempted murder. The woman properly could be convicted of attempted murder. Accordingly, conspiracy is not the most serious crime of which she could be convicted.

A

A woman promised to pay $10,000 to a hit man if he would kill her neighbor in any manner that could not be traced to her. The hit man bought a gun and watched the neighbor’s house for an opportunity to shoot him. One evening, unaware of the hit man’s presence, the neighbor tripped as he was walking toward his house, falling and hitting his head against the front steps. Believing that the neighbor was unconscious, the hit man ran over to him and shot him twice in the chest. When the woman learned of the neighbor’s death, she paid the hit man $10,000. A medical examiner determined that the neighbor was already dead when the hit man shot him. The crimes below are listed in descending order of seriousness. What is the most serious crime of which the woman properly could be convicted? Attempted murder.

How well did you know this?
1
Not at all
2
3
4
5
Perfectly
25
Q

The law does not trigger heightened judicial scrutiny, because it neither classifies regulatory subjects on a constitutionally suspect basis nor unduly burdens the exercise of a fundamental right. The appropriate constitutional standard of review therefore is whether the law is rationally related to a legitimate government interest. The apparent legislative judgment that diagnostic centers not affiliated with hospitals would be less reliable than hospitals is rational, regardless of whether it is in fact correct.

A

In one state, certain advanced diagnostic medical technologies were located only in hospitals, where they provided a major source of revenue. In many other states, such technologies were also available at “diagnostic centers” that were not affiliated with hospitals. A group of physicians announced its plan to immediately open in the state a diagnostic center that would not be affiliated with a hospital. The state hospital association argued to the state legislature that only hospitals could reliably handle advanced diagnostic medical technologies. The legislature then enacted a law prohibiting the operation in the state of diagnostic centers that were not affiliated with hospitals. The group of physicians filed suit challenging the constitutionality of the state law. What action should the court take? Uphold the law, because the legislature could rationally believe that diagnostic centers not affiliated with hospitals would be less reliable than hospitals.

How well did you know this?
1
Not at all
2
3
4
5
Perfectly
26
Q

If a party destroys evidence, it is proper for the jury to draw an inference that the evidence was adverse to that party’s case. It is also proper for the jury to draw an adverse inference in a civil case from a party’s assertion of the privilege against self-incrimination. Thus, the court should allow the question to be asked, because it is proper regardless of how the defendant responds.

A

A plaintiff sued an individual defendant for injuries suffered in a collision between the plaintiff’s car and the defendant’s truck while the defendant’s employee was driving the truck. The plaintiff sought discovery of any accident report the employee might have made to the defendant, but the defendant responded that no such report existed. Before trial, the defendant moved to preclude the plaintiff from asking the defendant in the presence of the jury whether he had destroyed such a report, because the defendant would then invoke his privilege against self-incrimination. Should the court allow the plaintiff to ask the defendant about the destruction of the report? Yes, because the defendant’s destruction of the report would serve as the basis of an inference adverse to the defendant. NOT, because a party in a civil action may not invoke the privilege against self-incrimination. - The privilege against self-incrimination may be asserted in both civil and criminal cases so long as the statement made in response to the question posed could tend to incriminate the person in a criminal prosecution.

How well did you know this?
1
Not at all
2
3
4
5
Perfectly
27
Q

Rule 12 of the Federal Rules of Civil Procedure (FRCP) provides that if a pre-answer motion asserting any waivable defense is filed, all waivable defenses must be raised in the same motion. After filing a pre-answer motion, a defendant is not permitted to make another pre-answer motion asserting any waivable defenses. See FRCP 12(g)(2); 12(h)(1). Here, the defendant filed a pre-answer motion, which was denied by the court. The defendant’s failure to raise the defense of insufficient service of process in the previously filed pre-answer motion effectively waived the defendant’s right to raise that defense.

A

A plaintiff brought a diversity action against a defendant in federal court. The plaintiff served the defendant with a copy of the summons and complaint by leaving a copy of the pleadings at the defendant’s home with the defendant’s housemaid, just as she was leaving to go home at the end of her shift. The complaint asserted negligence as the grounds for relief, and stated some facts as a basis for the claim, but the defendant had no recollection of being involved in any of the stated facts and was confused by the pleadings. The defendant filed a pretrial motion to dismiss for failure to state a claim or, alternatively, for a more definite statement. The court denied the motion. Thereafter, the defendant immediately, without filing an answer, moved to dismiss for insufficient service of process. Is the defendant’s motion to dismiss likely to succeed? No, because the defendant waived the opportunity to raise an insufficient service of process defense.

How well did you know this?
1
Not at all
2
3
4
5
Perfectly
28
Q

Lack of access may render title unmarketable under a contract of sale; however, the time to challenge marketable title is prior to the acceptance of the deed. Under the doctrine of merger, the remedy, if any, would be under one of the title covenants in the deed. Lack of access does not violate any of the title covenants. The colleague received the title the niece said she had. No one had a superior title and thus the covenants of seisin, right to convey, quiet enjoyment, and general warranty were not breached. The covenant against encumbrances provides protection for interests held by third parties such as easements for access. The land was not subject to an express easement nor may any easement be implied based on either prior use or necessity because the lands were never held in common ownership. The statute of frauds does require that an agreement to sell land be in writing. Nonetheless, if the parties have both fully performed under an oral contract, the relationship is the same as if the parties had fully complied with the statute initially. It is too late for the colleague, having accepted the deed, to now assert that the statute of frauds invalidates the oral agreement.

A

A niece inherited vacant land from her uncle. She lived in a distant state and decided to sell the land to a colleague who was interested in purchasing the land as an investment. They orally agreed upon a price, and, at the colleague’s insistence, the niece agreed to provide him with a warranty deed without any exceptions. The price was paid, the warranty deed was delivered, and the deed was promptly recorded. Neither the niece nor the colleague had, at that point, ever seen the land. After recording the deed, the colleague visited the land for the first time and discovered that it had no access to any public right-of-way and that none of the surrounding lands had ever been held in common ownership with any previous owner of the land. The colleague sued the niece for damages. For whom will the court find? The niece, because no title covenants were breached

How well did you know this?
1
Not at all
2
3
4
5
Perfectly
29
Q

This evidence has some probative value because it links the knife in the defendant’s possession to the type of knife that could have caused the victim’s wound. The evidence is not very strong, because other knives could also have caused the wound. But how much weight to give to the evidence is a decision for the jury. Rule 401 of the Federal Rules of Evidence requires only that evidence have any tendency to make the existence of any fact of consequence more or less probable than it would be without the evidence. Thus, to be relevant, evidence need only have some probative value in establishing a fact.

A

In a prosecution for aggravated battery, a police officer testified that when he arrested the defendant, he took a knife from the defendant and delivered it to the medical examiner. The medical examiner testified that the knife blade was consistent with the victim’s wound but admitted on cross-examination that any number of other knives could also have caused the wound. Should the judge grant a motion to strike the medical examiner’s testimony? No, because the probative worth of this evidence is for the jury to assess.

How well did you know this?
1
Not at all
2
3
4
5
Perfectly
30
Q

This statement is admissible both to impeach the defendant’s testimony as a prior inconsistent statement and as substantive evidence, because it is an admission of a party-opponent under Rule 801(d)(2)(A) of the Federal Rules of Evidence.

A

A defendant was charged with perjury for having falsely testified in an earlier civil case that he knew nothing about a business fraud. In the perjury trial, the defendant again testified that he knew nothing about the business fraud. In rebuttal, the prosecutor has called a witness to testify that after the civil trial was over, the defendant admitted to the witness privately that he had known about the fraud. Is the witness’s testimony in the perjury trial admissible? Yes, both to impeach the defendant’s testimony and as substantive evidence of the perjury.

How well did you know this?
1
Not at all
2
3
4
5
Perfectly
31
Q

The man is claiming a right to use a portion of the neighbor’s land, which is an easement. An easement by prescription requires that the use be without the owner’s permission for the requisite period of time. The man has used the path for the past 15 years without the neighbor’s permission. His use was open and notorious in that the neighbor could have seen him, it was continuous and without interruption by the neighbor, and it was actual. An easement acquired by prescription need not be exclusive. With an easement, the owner may make any use of the easement area that does not interfere with the use made by the easement holder, unless the easement is expressly noted as exclusive. The use by the neighbor’s tenants, the hunters, did not interfere with the man’s use, nor did his use interfere with theirs.

A

A man contacted his lawyer regarding his right to use a path that was on his neighbor’s vacant land. Fifteen years ago, after part of a path located on his land and connecting his cabin to the public highway washed out, the man cleared a small part of his neighbor’s land and rerouted a section of the path through the neighbor’s land. Twelve years ago, the neighbor leased her land to some hunters. For the next 12 years, the hunters and the man who had rerouted the path used the path for access to the highway. A month ago, the neighbor discovered that part of the path was on her land. The neighbor told the man that she had not given him permission to cross her land and that she would be closing the rerouted path after 90 days. The man’s land and the neighbor’s land have never been in common ownership. The period of time necessary to acquire rights by prescription in the jurisdiction is 10 years. The period of time necessary to acquire title by adverse possession in the jurisdiction is 10 years. What should the lawyer tell the man concerning his right to use the rerouted path on the neighbor’s land? The man has an easement by prescription to use the path.

How well did you know this?
1
Not at all
2
3
4
5
Perfectly
32
Q

The scope of cross-examination for all witnesses is controlled by Rule 611(b) of the Federal Rules of Evidence. (b) Scope of Cross-Examination. Cross-examination should not go beyond the subject matter of the direct examination and matters affecting the witness’s credibility. The court may allow inquiry into additional matters as if on direct examination.

A

A plaintiff sued a defendant for injuries allegedly suffered when he slipped and fell on the defendant’s business property. At trial, without asking that the defendant’s property manager be declared a hostile witness, the plaintiff called him solely to establish that the defendant was the owner of the property where the plaintiff fell. On cross-examination of the manager, the defendant’s attorney sought to establish that the defendant had taken reasonable precautions to make the property safe for business invitees. Should the defendant’s cross-examination of the manager be permitted over the plaintiff’s objection? No, because cross-examination should be limited to the subject matter of the direct examination and matters affecting the credibility of the witness.

How well did you know this?
1
Not at all
2
3
4
5
Perfectly
33
Q

Under the theory of res ipsa loquitur, the jury can infer negligence where an accident would not ordinarily have occurred in the absence of negligence and the defendant is responsible for the instrumentality that inflicted the injury. The supermarket is responsible for maintenance of the door under the lease and also is legally responsible for negligence as a land occupier because it invited the public to enter the store to shop.

A

As a shopper was leaving a supermarket, an automatic door that should have opened outward opened inward, striking and breaking the shopper’s nose. The owner of the building had installed the automatic door. The lease, pursuant to which the supermarket occupied the building, provided that the supermarket was responsible for all maintenance of the premises. The shopper sued the supermarket. At trial, neither the shopper nor the supermarket offered any testimony, expert or otherwise, as to why the door had opened inward. At the close of evidence, both the shopper and the supermarket moved for judgment as a matter of law. How should the trial judge rule? Submit the case to the jury, because on these facts negligence may be inferred.

How well did you know this?
1
Not at all
2
3
4
5
Perfectly
34
Q

The tort of intentional infliction of emotional distress allows recovery for personal injury despite the absence of physical injury or touching of the plaintiff. On these facts, the neighbor was aware that his conduct would cause severe emotional distress, and he could be held liable for the man’s emotional suffering, as well as for the value of the cat. The standard remedy in conversion is a forced sale, so the man could recover no more than $25, the value of the cat. An action establishing intentional infliction of emotional distress would result in a larger recovery.

A

A man owned a much-loved cat, worth about $25, that frequently trespassed on a neighbor’s property. The neighbor repeatedly asked the man to keep the cat on his own property, but the trespasses did not diminish. Aware of the man’s long-standing attachment to the cat, the neighbor killed the cat with a shotgun in full view of the man. As a consequence, the man suffered great emotional distress. In an action by the man against the neighbor, which of the following claims would be likely to result in the greatest monetary recovery? Intentional infliction of emotional distress.

How well did you know this?
1
Not at all
2
3
4
5
Perfectly
35
Q

In New York v. United States, 505 U.S. 144 (1992), the U.S. Supreme Court held that the concept of federalism embedded in the Tenth Amendment disables Congress from requiring states to enact laws or to administer federal law.

A

National statistics revealed a dramatic increase in the number of elementary and secondary school students bringing controlled substances to school for sale. In response, Congress enacted a statute requiring each state’s legislature to enact a law making it a crime for any person to sell, within 1,000 feet of any elementary or secondary school, any controlled substance that had previously been transported in interstate commerce. Is the federal statute constitutional? No, because Congress has no authority to require a state legislature to enact any specified legislation.

How well did you know this?
1
Not at all
2
3
4
5
Perfectly
36
Q

If a party moves for summary judgment, the nonmoving party must provide enough evidence to raise a minimally reasonable inference in his or her favor. See Wright, et. al., 10A Fed. Prac. & Proc. Civ. § 2725 (3d ed.). Here, the expert’s testimony that the video appears tampered with would meet that requirement. Do not want ambiguities - ambiguities are usually drawn in favor of the nonmoving party - so this answer is a no: Deny the motion, because ambiguities in the evidence are drawn in favor of the moving party.

A

A man brought a diversity action in federal court against a woman, seeking to recover for an injury he sustained at a youth lacrosse game when the woman hit him with a lacrosse stick. The only issue in the man’s action, which was based on a common law battery theory, was whether the woman actually hit him. After discovery, the man filed a motion for summary judgment. In support of his motion, he provided a witness’s cellphone video that clearly showed the woman hitting the man with a lacrosse stick. In opposition to the motion, the woman provided the sworn affidavit of an expert, who claimed that he would testify that the cellphone video appeared to have been tampered with. Assume that it is appropriate for the judge to consider the expert’s affidavit under Rule 56 of the Federal Rules of Civil Procedure. How should the court rule on the summary judgment motion? Deny the motion, because the woman submitted enough evidence to raise a minimally reasonable inference in her favor.

How well did you know this?
1
Not at all
2
3
4
5
Perfectly
37
Q

The power of the bookkeeper and the accountant to modify their duties by subsequent agreement was terminated when the clerk, an intended beneficiary of their contract, materially relied on their promised performance by purchasing the retirement home.

A

An accountant and a bookkeeper, as part of a contract dissolving their accounting business, agreed that each would contribute $100,000 to fund an annuity for a clerk who was a longtime employee of the business. The clerk’s position would be terminated at the dissolution, and he did not have a retirement plan. The accountant and the bookkeeper informed the clerk of their plan to fund an annuity for him. The clerk, confident about his financial future because of the promised annuity, purchased a retirement home. The accountant later contributed his $100,000 to fund the annuity, but the bookkeeper stated that he could afford to contribute only $50,000. The accountant agreed in writing that the bookkeeper should contribute only Does the clerk have a valid basis for an action against the bookkeeper for the unpaid $50,000? Yes, because the clerk’s reliance on the promised annuity prevented the parties from changing the terms of the contract.

How well did you know this?
1
Not at all
2
3
4
5
Perfectly
38
Q

The friend does not have an enforceable lien. The friend did have a lien on the lot when the investor granted the friend a mortgage. The friend, however, did not record the mortgage. The investor then sold the lot to the buyer. The buyer had no actual notice of the mortgage to the friend. The buyer had no notice based on possession because the lot was vacant. The buyer had no constructive notice of the mortgage because the mortgage to the friend had not been recorded when the buyer received title. The lot is located in a notice jurisdiction. Thus, the buyer took the lot free of any prior unrecorded interests. The buyer was an innocent purchaser for value at the time the buyer received title. Later notice to the buyer and the later recording of the friend’s mortgage are irrelevant.

A

Five years ago, an investor who owned a vacant lot in a residential area borrowed $25,000 from a friend and gave the friend a note for $25,000 due in five years, secured by a mortgage on the lot. The friend neglected to record the mortgage. The fair market value of the lot was then $25,000. Three years ago, the investor discovered that the friend had not recorded his mortgage and in consideration of $50,000 conveyed the lot to a buyer. The fair market value of the lot was then $50,000. The buyer knew nothing of the friend’s mortgage. One month thereafter, the friend discovered the sale to the buyer, recorded his $25,000 mortgage, and notified the buyer that he held a $25,000 mortgage on the lot. Two years ago, the buyer needed funds. Although she told her bank of the mortgage claimed by the investor’s friend, the bank loaned her $15,000, and she gave the bank a note for $15,000 due in two years secured by a mortgage on the lot. The bank promptly recorded the mortgage. At that time, the fair market value of the lot was $75,000. The recording act of the jurisdiction provides: “No conveyance or mortgage of real property shall be good against subsequent purchasers for value and without notice unless the same be recorded according to law.” Both notes are now due, and both the investor and the buyer have refused to pay. The lot is now worth only $50,000. What are the rights of the investor’s friend and the bank in the lot? Only the bank’s mortgage is an enforceable lien, because the buyer was an innocent purchaser for value.

How well did you know this?
1
Not at all
2
3
4
5
Perfectly
39
Q

A seller can treat a buyer’s failure to specify as a breach only if the buyer’s failure to specify materially impacts the seller’s performance. The seller had an available supply of candy bars and had entered into no new contracts. These facts support the conclusion that the buyer’s failure to select did not materially impact the seller’s performance. Therefore the seller unjustifiably refused to accept the buyer’s selection of goods. UCC § 2-311.

A

A seller and a buyer entered into a written agreement providing that the seller was to deliver 1,000 cases of candy bars to the buyer during the months of May and June. Under the agreement, the buyer was obligated to make a selection by March 1 of the quantities of the various candy bars to be delivered under the contract. The buyer did not make the selection by March 1, and on March 2, the seller notified the buyer that because of the buyer’s failure to select, the seller would not deliver the candy bars. The seller had all of the necessary candy bars on hand on March 1 and made no additional sales or purchases on March 1 or March 2. On March 2, after receiving the seller’s notice that it would not perform, the buyer notified the seller of its selection and insisted that the seller perform. The seller refused. If the buyer sues the seller for breach of contract, is the buyer likely to prevail? Yes, because the delay of one day in making the selection did not have a material effect on the seller.

How well did you know this?
1
Not at all
2
3
4
5
Perfectly
40
Q

Fifteen years ago, each of the parties granted a reciprocal right of first refusal (or a preemptive right) to the other. A right of first refusal is a conditional option. It provides that if the owner ever decides to sell the property, the person or entity holding the right of first refusal has the right to purchase the property on specified terms. In this case, the purchase price was to be set by three qualified expert independent real estate appraisers and was thus fair. These rights of first refusal, however, violate the common law Rule Against Perpetuities. The right to purchase is triggered by the decision to sell the land. In this case, that decision might occur more than 21 years after a life in being at the time the right was granted. Thus, under the common law, this right of first refusal is struck ab initio. The question notes that the common law Rule Against Perpetuities is unmodified in this jurisdiction. Thus, there are no applicable statutory reforms to the rule. And because the daughter prevails, there must be no applicable exceptions to the common law rule.

A

A grantor owned two tracts of land, one of 15 acres and another of 5 acres. The two tracts were a mile apart. Fifteen years ago, the grantor conveyed the smaller tract to a grantee. The grantor retained the larger tract. The deed to the grantee contained, in addition to proper legal descriptions of both properties and identifications of the parties, the following language: “I, the grantor, bind myself and my heirs and assigns that in the event that the larger tract that I now retain is ever offered for sale, I will notify the grantee and his heirs and assigns in writing, and the grantee and his heirs and assigns shall have the right to purchase the larger tract for its fair market value as determined by a board consisting of three qualified expert independent real estate appraisers.” With appropriate references to the other property and the parties, there followed a reciprocal provision that conferred upon the grantor and her heirs and assigns a similar right to purchase the smaller tract, purportedly binding the grantee and his heirs and assigns. Ten years ago, a corporation acquired the larger tract from the grantor. At that time, the grantee had no interest in acquiring the larger tract and by an appropriate written document released any interest he or his heirs or assigns might have had in the larger tract. Last year, the grantee died. The smaller tract passed by the grantee’s will to his daughter. She has decided to sell the smaller tract. However, because she believes that the corporation has been a very poor steward of the larger tract, she refuses to sell the smaller tract to the corporation even though she has offered it for sale in the local real estate market. The corporation has brought an appropriate action for specific performance of the right of first refusal after taking all of the necessary preliminary steps in its effort to exercise its right to purchase the smaller tract. The daughter has asserted all possible defenses. The common law Rule Against Perpetuities is unmodified in the jurisdiction, and there are no applicable statutes. If the court rules for the daughter, what will be the likely reason? The provision setting out the right to purchase violates the Rule Against Perpetuities.

How well did you know this?
1
Not at all
2
3
4
5
Perfectly
41
Q

Use and derivative use immunity sufficiently protects the constitutional privilege against self-incrimination in this situation. derivative use immunity is more common (used by both state and federal prosecutors) and narrower than transactional immunity. It prevents the prosecution from using the witness’s statements (“use”) or any evidence derived from those statements (“derivative use”) against the witness in a criminal prosecution.

A

A state grand jury investigating a murder learned that the key suspect might have kept a diary. The grand jury issued a subpoena duces tecum requiring the suspect to produce any diary. The subpoena made clear that the grand jury was seeking only the diary and not any testimony from the suspect. The suspect refused to produce the diary, citing his Fifth Amendment privilege against self-incrimination. Under what circumstances, if any, may the grand jury compel production of the diary over the suspect’s assertion of his Fifth Amendment privilege? It may compel production only if the suspect is granted use and derivative use immunity from the act of production.

How well did you know this?
1
Not at all
2
3
4
5
Perfectly
42
Q

The court must have applied strict liability to convict the clerk (who did not act knowingly, and arguably not even negligently) and vicarious liability to convict the store owner for the sale by the clerk.

A

A state statute provides: “The sale of an alcoholic beverage to any person under the age of 21 is a misdemeanor.” A woman who was 20 years old, but who looked older and who had a very convincing fake driver’s license indicating that she was 24, entered a convenience store, picked up a six-pack of beer, and placed the beer on the counter. The store clerk, after examining the driver’s license, rang up the purchase. Both the clerk and the store owner have been charged with violating the state statute. If the court finds both the clerk and the store owner guilty, what standard of liability must the court have interpreted the statute to impose? Both strict and vicarious liability.

How well did you know this?
1
Not at all
2
3
4
5
Perfectly
43
Q

Pursuant to 28 U.S.C. § 1441(b)(2), where a lawsuit is removable only on the basis of diversity jurisdiction, it cannot be removed if any defendant in the case is a citizen of the state where the lawsuit was brought. This is known as the forum-defendant rule. Here, the only basis for removal is diversity jurisdiction, and the second defendant is a citizen of the state where the complaint was filed (Oklahoma). Accordingly, the forum-defendant rule prevents the defendant from removing the case from state court in Oklahoma to federal court in Oklahoma. Answer option C is incorrect because a plaintiff’s consent is not required to remove a case from state court to federal court. A defendant only needs the consent of any other defendants that have been served.

A

A plaintiff filed a complaint in Oklahoma state court against two defendants, alleging a state-law claim of negligence and seeking over $100,000 in damages. The plaintiff is a citizen of Colorado, the first defendant is a citizen of Iowa, and the second defendant is a citizen of Oklahoma. Twenty-five days after being served, the second defendant, with the consent of the first defendant, filed a notice of removal in federal district court in Oklahoma, on the basis of diversity jurisdiction. What is the plaintiff’s best argument in seeking to remand the case to state court? The second defendant failed to get the plaintiff’s consent to remove the case to federal court.

How well did you know this?
1
Not at all
2
3
4
5
Perfectly
44
Q

An assignee succeeds to a contract as the contract stands at the time of the assignment. In this case, the parties had modified the contract as to when the payments were due. (Note that there was consideration for the promise to accept payments later; the consideration was the debtor’s promise to make future payments by cashier’s check.) Accordingly, the debtor can insist that the payments be due on the fifth of each month. no such rule that contract modification is not binding on an assignee who had no knowledge of the modification.

A

On January 5, a creditor loaned $1,000 to a debtor under a contract calling for the debtor to repay the loan at the rate of $100 per month payable on the first day of each month. On February 1, at the debtor’s request, the creditor agreed to permit payment on February 5. On March 1, the debtor requested a similar time extension and the creditor replied, “Don’t bother me each month. Just change the date of payment to the fifth of the month. But you must now make the payments by cashier’s check.” The debtor said, “Okay,” and made payments on March 5 and April 5 by cashier’s check. On April 6, the creditor sold the loan contract to a bank but did not tell the bank about the agreement permitting payments on the fifth of the month. On April 6, the bank wrote to the debtor: “Your debt to [the creditor] has been assigned to us. We hereby inform you that all payments must be made on the first day of the month.” Can the debtor justifiably insist that the payment date for the rest of the installments is the fifth of each month? Yes, because the creditor could assign to the bank only those rights the creditor had in the contract at the time of the assignment.

How well did you know this?
1
Not at all
2
3
4
5
Perfectly
45
Q

The Supreme Court may not review a judgment by the highest court of a state if that judgment is supported entirely by state law and is wholly independent of the interpretation and application of federal law. In this case, although the defendant claimed a violation of the Sixth Amendment to the U.S. Constitution, the state’s highest court based its decision entirely on the state constitution without addressing the federal constitutional issue.

A

A state constitution provides that in every criminal trial “the accused shall have the right to confront all witnesses against him face to face.” A defendant was convicted in state court of child abuse based on testimony from a six-year-old child. The child testified while she was seated behind one-way glass, which allowed the defendant to see the child but did not allow the child to see the defendant. The defendant appealed to the state’s highest court, claiming that the inability of the child to see the defendant while she testified violated both the United States Constitution and the state constitution. Without addressing the federal constitutional issue, the state’s highest court reversed the defendant’s conviction and ordered a new trial. The court held that “the constitution of this state is clear, and it requires that while testifying in a criminal trial, a witness must be able to see the defendant.” The state petitioned the United States Supreme Court for a writ of certiorari. On which ground should the United States Supreme Court DENY the state’s petition? The decision of the state’s highest court was based on an adequate and independent state ground.

How well did you know this?
1
Not at all
2
3
4
5
Perfectly
46
Q

vidence of the defendant’s prior conviction is certainly relevant to the current charge of being a felon in possession of a firearm, because the previous conviction is the predicate for the offense with which the defendant is charged. Accordingly, the relevance of the evidence is not in question and would support admission rather than exclusion of the evidence. The concepts listed in the remaining answer options—probative value, unfair prejudice, and waste of time—all would lend more support to a decision to exclude the evidence.

A

A defendant was on trial for being a felon in possession of a firearm. The defendant wished to stipulate to the fact that he was previously convicted of a felony, rather than have the jury hear evidence about his prior felony conviction for kidnapping. The prosecution nevertheless sought to introduce the details of the prior conviction. Which of the following is the WEAKEST argument for excluding the evidence? The prior conviction is not relevant to the current charge.

How well did you know this?
1
Not at all
2
3
4
5
Perfectly
47
Q

An easement confers a limited right to use or enjoy the land of another. An affirmative easement gives the holder the right to enter the servient estate to do something (e.g., use a walking path across the servient estate). A negative easement gives the holder the right to prevent a specified use of the servient estate (e.g., blocking light or air). An easement may be created by express grant in a written instrument; by reservation as part of a conveyance; by necessity; or by prescription where a trespasser openly, notoriously, and adversely uses the servient estate for a prescribed period of time. Only affirmative easements can arise from prescription. Here, the astronomer has not entered the neighbor’s property but seeks to prevent the neighbor from blocking his view of the sky, suggesting a negative easement. However, there is no indication in the facts that the neighbor ever expressly granted such an easement or that it was otherwise reserved in a previous conveyance. Because a negative easement may not be created by prescription, it is irrelevant whether the astronomer has been stargazing openly, notoriously, and adversely for the time period required to obtain an easement by prescription. The astronomer has no easement and will not be successful in halting the neighbor’s construction.

A

An astronomer used a number of telescopes to track the position of planets and stars from his home’s deck. The astronomer engaged in this stargazing activity four to five nights a week for 15 years, during which time his neighbor regularly saw him using the telescopes and frequently came outside to chat with him about his findings. Recently, the neighbor began construction on a second story for his house, which had always been a single-story residence. The second story will block most of the astronomer’s view of the sky from his deck. The period for obtaining an easement by prescription in the jurisdiction is 10 years. If the astronomer seeks to halt the neighbor’s construction project, is the astronomer likely to prevail? No, because a negative easement may not be created by prescription.

How well did you know this?
1
Not at all
2
3
4
5
Perfectly
48
Q
A
How well did you know this?
1
Not at all
2
3
4
5
Perfectly
49
Q

An easement confers a limited right to use or enjoy the land of another. An affirmative easement gives the holder the right to enter the servient estate to do something (e.g., use a walking path across the servient estate). A negative easement gives the holder the right to prevent a specified use of the servient estate (e.g., blocking light or air). An easement may be created by express grant in a written instrument; by reservation as part of a conveyance; by necessity; or by prescription where a trespasser openly, notoriously, and adversely uses the servient estate for a prescribed period of time. Only affirmative easements can arise from prescription. Here, the neighbor has entered the investor’s property regularly for the prescribed period, and the investor knew of the neighbor’s ongoing use. Although the investor never expressly granted the neighbor an easement, the neighbor has been openly, notoriously, and adversely using the pier for the time period required to obtain an easement by prescription.

A

An investor owned a lakefront vacation home that she visited two to three days per month. During one of her visits, the investor saw a neighbor fishing on her pier. She was annoyed but said nothing to the neighbor. Over the course of the next ten years, the investor occasionally found evidence of her neighbor’s continued use of the pier, and the neighbor occasionally emailed her a photo of himself on the pier with one of his larger catches. The period for obtaining an easement by prescription in the jurisdiction is 10 years.

Is the neighbor entitled to continue using the investor’s pier?

Yes, because the neighbor has acquired an affirmative easement by prescription.

How well did you know this?
1
Not at all
2
3
4
5
Perfectly
50
Q

Although a defendant’s statement obtained in violation of his Sixth Amendment rights may generally be used to impeach the defendant’s inconsistent testimony at trial, this impeachment exception does not extend to the impeachment of other defense witnesses or to the rehabilitation of prosecution witnesses. See James v. Illinois, 493 U.S. 307, 310 (1990). Answer option C is incorrect because, although the confession is inadmissible in the given scenario, unlawfully obtained confessions may be used under certain circumstances, such as to impeach a defendant’s own inconsistent testimony.

A

A criminal defendant was arrested on a charge of bank robbery. At the police station following the defendant’s arrest, officers interrogated the defendant without advising him of his right to counsel. The defendant answered the officers’ questions truthfully and confessed to the burglary. Defense counsel later moved to suppress the confession on Sixth Amendment grounds, and the court granted the motion. At trial, a witness for the prosecution testified that she had been at the bank at the time of the robbery, and identified the defendant as the robber. On cross-examination, defense counsel elicited damaging information from the witness regarding inconsistencies in where she was standing at the time of the robbery, her line of sight to the robber, and her recollection of other details relating to the scene of the crime. On redirect, the prosecution sought to rehabilitate the witness’s testimony with evidence of the defendant’s confession. Defense counsel objected.

Will the court allow the prosecution to use the defendant’s confession to rehabilitate the witness’s testimony?

No, because an unlawfully obtained confession may only be used to impeach a defendant’s own testimony.

NOT - No, because an unlawfully obtained confession may not be introduced at trial for any purpose.

How well did you know this?
1
Not at all
2
3
4
5
Perfectly
51
Q

Because the pedestrian was in the path of the truck, he was under a direct physical threat from the driver’s negligence. He could recover for the emotional distress that he suffered as a result of his fear for his own safety, and many courts would also allow him to recover for all other emotional distress that he suffered in connection with the event.

A

A pedestrian was crossing a street in a crosswalk when a woman walking just ahead of him was hit by a truck. The pedestrian, who had jumped out of the way of the truck, administered CPR to the woman, who was a stranger. The woman bled profusely, and the pedestrian was covered in blood. The woman died in the ambulance on the way to the hospital. The pedestrian became very depressed immediately after the incident and developed physical symptoms as a result of his emotional distress. The pedestrian has brought an action against the driver of the truck for negligent infliction of emotional distress. In her defense, the driver asserts that she should not be held liable, because the pedestrian’s emotional distress and resulting physical symptoms are not compensable.

What is the strongest argument that the pedestrian can make in response to the driver’s defense?

The pedestrian was in the zone of danger.

How well did you know this?
1
Not at all
2
3
4
5
Perfectly
52
Q

Rule 15(a) of the Federal Rules of Civil Procedure (FRCP) provides that a court should give liberal leave to amend absent limited circumstances where denial of leave to amend is justified (e.g., if the motion is made in bad faith, would cause undue delay, or would prejudice the opposing party). Here, there is no indication of special circumstances that would warrant a denial of the plaintiff’s motion to amend; accordingly, the court will likely grant the plaintiff’s motion.

A

A prison inmate filed a pro se civil rights complaint alleging that the defendant prison guards had beaten him. The inmate did not identify any specific guard who was involved and did not provide dates or details of where the beatings took place. The defendants moved to dismiss the complaint, arguing that the complaint and its allegations were insufficiently pled. The inmate moved to amend his complaint.

Is the court likely to grant the defendants’ motion to dismiss?

No, because the court must be liberal in allowing the parties to amend their pleadings where there is a possibility that amendment will be fruitful.

NOT - Yes, because a plaintiff’s complaint is only properly amended prior to the defendant’s answer or pre-answer motion.

How well did you know this?
1
Not at all
2
3
4
5
Perfectly
53
Q

Under the UCC’s parol evidence rule, a merger clause does not conclusively establish that an agreement is completely integrated. Moreover, a finding that an agreement is completely integrated does not necessarily bar the admission of extrinsic evidence. Although extrinsic evidence is inadmissible to supplement or contradict the express terms of a completely integrated agreement, such evidence is admissible to explain the terms of an agreement. In this case, evidence of the parties’ discussions during their negotiations is admissible to aid in explaining what amount they intended for the trade-in and whether they intended trade-in allowance to include an assignment of the buyer’s claim against her insurance company.

A

A buyer purchased a new car from a dealer under a written contract that provided that the price of the car was $20,000 and that the buyer would receive a “trade-in allowance of $7,000 for the buyer’s old car.” The old car had recently been damaged in an accident. The contract contained a merger clause stating: “This writing constitutes the entire agreement of the parties, and there are no other understandings or agreements not set forth herein.” When the buyer took possession of the new car, she delivered the old car to the dealer. At that time, the dealer claimed that the trade-in allowance included an assignment of the buyer’s claim against her insurance company for damage to the old car. The buyer refused to provide the assignment. The dealer sued the buyer to recover the insurance payment. The dealer has offered evidence that the parties agreed during their negotiations for the new car that the dealer was entitled to the insurance payment.

Should the court admit this evidence?

Yes, because the merger clause does not bar evidence to explain what the parties meant by trade-in allowance.

54
Q

A seller may disclaim any duty to disclose defects if the disclaimer is sufficiently clear and specific. In this case, the contract specifically noted that the house was being sold “as is.” The woman made no misrepresentations regarding the condition of the house. There are no statutes that might require an owner-occupier to disclose known defects, and in any case the woman inherited the house and had never visited or lived in it. In addition, this is not the sale of a new house by a builder/seller, which may impose a warranty of habitability.A warranty deed provides remedies for breaches of title matters.

A

A woman inherited a house from a distant relative. The woman had never visited the house, which was located in another state, and did not want to own it. Upon learning this, a man who lived next door to the house called the woman and asked to buy the house. The woman agreed, provided that the house was sold “as is.” The man agreed, and the woman conveyed the house to the man by a warranty deed. The man had purchased the house for investment purposes, intending to rent it out while continuing to live next door. After the sale, the man started to renovate the house and discovered serious termite damage. The man sued the woman for breach of contract. There are no applicable statutes.

How should the court rule?

  • For the woman, because the man planned to change the use of the house for investment purposes.
  • For the woman, because she sold the house “as is.”
  • For the man, because of the doctrine of caveat emptor.
  • For the man, because he received a warranty deed.
55
Q
  • Rule 615 provides that if a party moves to exclude prospective witnesses before they testify, “the court must order witnesses excluded so they cannot hear other witnesses’ testimony.”
  • Under Rule 612, the trial court has discretion to order a party to produce for the adversary a writing used before trial to refresh the memory of a witness called by the party, but the court is not required to do so.
  • Rule 612, the trial court has discretion to order a party to produce for the adversary a writing used before trial to refresh the memory of a witness called by the party, but the court is not required to do so.
  • Under Rule 601, all witnesses are presumed to be competent. The man can use this evidence to impeach the witness when the witness testifies.
A

A man suffered a broken jaw in a fight with a neighbor that took place when they were both spectators at a soccer match.

If the man sues the neighbor for personal injury damages, which of the following actions must the trial court take if requested by the man?

  • Prevent the neighbor’s principal eyewitness from testifying, upon a showing that six years ago the witness was convicted of perjury and the conviction has not been the subject of a pardon or annulment.
  • Refuse to let the neighbor cross-examine the man’s medical expert on matters not covered on direct examination of the expert.
  • Exclude nonparty eyewitnesses from the courtroom during the testimony of other witnesses.
  • Require the production of a writing used before trial to refresh a witness’s memory.
56
Q

The express easement granted to the woman was terminated by merger when she reconveyed her land to the man, who again became the common owner of both tracts. The man then conveyed the rear half of the land to the investor, whose land is now landlocked. An easement by necessity is implied for the benefit of the landlocked parcel at the moment of severance of the common ownership.

A

A man owned a large tract of land that had frontage on a public highway. The land had no access to any other road. Fifteen years ago, the man conveyed the rear half of the land to a woman and at the same time conveyed an express easement to the woman that provided access from her land across his retained land to the public highway. The woman used the easement until she reconveyed the land back to the man 10 years ago. The deed to the man made no reference to the easement. Five years ago, the man again conveyed the rear half of the land, this time to an investor in a deed that made no reference to any easement to the public highway. Recently, the man told the investor that he could no longer cross the man’s land for access to the public highway. A neighbor has told the investor that he can use her land for access to another public road “for a price.” The investor has sued the man for the right to cross the man’s land to the public highway.

For whom will the court likely decide?

  • The investor, because an easement will be implied
  • The man, because the express easement was terminated by the reconveyance.
  • The man, because the investor can reasonably acquire another means of access to a public road.
  • The investor, because the man is estopped by his grant of an easement to the woman.
57
Q

The seller’s sale of the car to the third party was an anticipatory repudiation that gave the buyer an immediate claim for breach of contract. Under UCC Article 2, a seller’s tender of delivery of goods and a buyer’s tender of payment are concurrent conditions of exchange. Therefore, the buyer and the seller were obligated to simultaneously tender their respective performances. However, because the seller breached by anticipatory repudiation, the buyer’s performance obligation was discharged. Accordingly, the buyer has a claim for breach of contract, even though she did not tender performance.

A

A buyer agreed in writing to purchase a car from a seller for $15,000, with the price to be paid on a specified date at the seller’s home. The contract provided, and both parties intended, that time was of the essence. Before the specified date, however, the seller sold the car to a third party for $18,000. On the specified date, the buyer arrived at the seller’s home prepared to tender payment. The seller was not there, so the buyer called the seller to ask where he was. The seller then told the buyer that he had sold the car to the third party.

If the buyer sues the seller for breach of contract, will the buyer be likely to prevail?

  • Yes, because the seller anticipatorily repudiated the contract when he sold the car to the third party.
  • Yes, because the seller did not inform the buyer of his repudiation.
  • No, because the buyer did not tender her performance on the specified date.
  • No, because the contractual obligations were discharged on the ground of impossibility.
58
Q

Under Rule 15(a)(1) of the Federal Rules of Civil Procedure (FRCP), a party may amend its pleading “once as a matter of course” before trial within either: (1) 21 days after serving its pleading, or (2) if the pleading sought to be amended is one that requires the opposing party’s response, then 21 days after the responsive pleading is served or 21 days after service of a motion under Rule 12(b), (e), or (f), whichever is earlier. Here, because the defendant’s counterclaim is considered a pleading to which a response is required from the plaintiff, the defendant can only amend his counterclaim before trial “as a matter of course” within 21 days from service of the plaintiff’s reply. The defendant realized his mistake four weeks after plaintiff’s reply was served and then belatedly sought to amend his counterclaim thereafter. Because more than 21 days have passed since the service of the plaintiff’s reply, the defendant’s time to amend as a matter of course has expired. Accordingly, the defendant needs leave of court or consent from the plaintiff in order to amend his counterclaim.

A

A plaintiff brought a diversity action in federal court, claiming that she was injured when the defendant negligently crashed into her with his car. The defendant counterclaimed that the plaintiff had breached a contract with the defendant for the purchase of the defendant’s property. The plaintiff answered the counterclaim within 10 days, denying the counterclaim’s allegations. Four weeks after the plaintiff had served her answer to the defendant’s counterclaim, the defendant realized that the purchase agreement was unenforceable under the statute of frauds. The defendant would like to change his counterclaim theory to negligent misrepresentation, based on the plaintiff’s verbal agreement to purchase the defendant’s property.

Can the defendant file an amended counterclaim without leave of court and without the plaintiff’s consent?

  • No, because more than 21 days have passed from the date of the plaintiff’s answer to the defendant’s counterclaim.
  • Yes, because the defendant can amend his counterclaim at least once as a matter of course.
  • Yes, because if the defendant files immediately, the amendment will be filed within 30 days after the plaintiff’s answer to the defendant’s counterclaim.
  • No, because the defendant can never amend a counterclaim without leave of court or the plaintiff’s consent.
59
Q

The killing was committed willfully with premeditation and deliberation. The killing cannot be justified as having been in self-defense, because the man was the clear aggressor who intentionally provoked the coworker so that he could shoot and kill him.

The killing cannot be justified as having been in self-defense, because the man intentionally provoked the coworker so that he could shoot and kill him. The man is guilty of first-degree murder, because he committed the killing willfully with premeditation and deliberation.

A

A state statute divides murder into degrees and defines murder in the first degree as murder committed willfully with premeditation and deliberation. The statute defines murder in the second degree as all other murder at common law and defines voluntary manslaughter as at common law. A man hated one of his coworkers. Upon learning that the coworker was at a neighbor’s house, the man grabbed his gun and went to the neighbor’s house hoping to provoke the coworker into attacking him so that he could then shoot the coworker. After arriving at the house, the man insulted the coworker and bragged that he had had sexual relations with the coworker’s wife two weeks earlier. This statement was not true, but it enraged the coworker, who grabbed a knife from the kitchen table and ran toward the man. The man then shot and killed the coworker.

What is the most serious homicide offense of which the man could properly be convicted?

  • Murder in the first degree.
  • Murder in the second degree.
  • Voluntary manslaughter, because he provoked the coworker.
  • No form of criminal homicide, because he acted in self-defense.
60
Q

The decedent’s statement of his medical history was made for the purpose of diagnosis and treatment, and it is clearly pertinent to the physician’s diagnosis and treatment. Therefore, it is admissible under Rule 803(4).

A

The beneficiary of a decedent’s life insurance policy has sued the life insurance company for the proceeds of the policy. At issue is the date when the decedent first experienced the heart problems that led to his death. The decedent’s primary care physician has testified at trial that the decedent had a routine checkup on February 15. The physician then identifies a photocopy of a questionnaire, provided by the physician and completed by the decedent on that date, in which the decedent wrote: “Yesterday afternoon I broke into a big sweat and my chest hurt for a while.” The beneficiary now offers the photocopy in evidence.

Should the court admit the photocopy?

Yes, as a statement for the purpose of obtaining medical treatment.

61
Q

The First Amendment applies to discretionary decisions of governments and government officials. The man is unlikely to prevail, but it is because the First and Fourteenth Amendments generally apply only to the actions of governments and government officials, not to the actions of privately owned companies such as the insurance company.

A

A state law imposed substantial regulations on insurance companies operating within the state with respect to their rates, cash reserves, and financial practices. A privately owned insurance company operating within the state advertised that it wanted to hire a new data processor. After reviewing applications for that position, the company hired a woman who appeared to be well qualified. The company refused to consider the application of a man who was better qualified than the woman, because he was known to have radical political views. The man sued the company, alleging only a violation of his federal constitutional right to freedom of expression.

Is the man likely to prevail?

No, because the company is not subject to the provisions of the First and Fourteenth Amendments.

NOT- No, because hiring decisions are wholly discretionary and thus are not governed by the First Amendment.

62
Q

The statute of frauds requires a contract for the sale of land to identify the parties, contain a description of the land, evidence an intent to buy and sell, recite (usually) a price term, and be signed by the party against whom enforcement is sought. The email messages here fulfill those requirements. Courts are liberal regarding the nature of a signature; it need only reflect an intent to authenticate the writing. Both the tenant’s and the landlord’s names were placed below the word signed, which adequately reflected their desire to be bound.

A

A landlord leased a building to a tenant for a term of six years. The lease complied with the statute of frauds and was not recorded. During the lease term, the tenant sent an email to the landlord that stated: “I hereby offer to purchase for $250,000 the building that I am now occupying under a six-year lease with you.” The tenant’s name was placed below the word “signed” on the message. In response, the landlord emailed the tenant: “That’s fine. We’ll close in 60 days.” The landlord’s name was placed below the word “signed” on the reply message. Sixty days later, the landlord refused to tender the deed to the building when the tenant tendered the $250,000 purchase price. The tenant has sued for specific performance.

Who is likely to prevail?

  • The landlord, because formation of an enforceable contract to convey the building could not occur until after the lease term expired.
  • The landlord, because the landlord’s email response did not contain a sufficient signature under the statute of frauds.
  • The tenant, because the email messages constitute an insufficient attornment of the lease.
  • The tenant, because the email messages constitute a sufficient memorandum under the statute of frauds.
63
Q

Because the contract authorizes the delivery of flour in separate lots to be separately accepted, the parties entered into an installment contract. UCC § 2-612 adopts a “substantial impairment” standard for determining whether a buyer can reject a particular installment or cancel the entire contract. A buyer can reject an installment if a nonconformity substantially impairs that installment and the nonconformity cannot be cured. Here the mill’s tender of less than the contracted-for quantity did not amount to a nonconformity that substantially impaired either the value of the 16th installment or the whole contract. The mill’s proposed cure, the delivery of the remaining 200 pounds on the following Monday, is sufficient given that the late delivery and the shortage will not significantly disrupt the bakery’s business. Accordingly, the bakery must accept the delivery of the tendered 800 pounds of flour but may deduct from the price any damages for losses resulting from the late delivery.

A

A mill and a bakery entered into a written contract that obligated the mill to deliver to the bakery 1,000 pounds of flour every Monday for 26 weeks at a specified price per pound. The mill delivered the proper quantity of flour in a timely manner for the first 15 weeks. However, the 16th delivery was tendered on a Tuesday, and amounted to only 800 pounds. The mill told the bakery that the 200-pound shortage would be made up on the delivery due the following Monday. The late delivery and the 200-pound shortage will not significantly disrupt the bakery’s operations.

How may the bakery legally respond to the nonconforming tender?

Accept the 800 pounds tendered, but notify the mill that the bakery will deduct from the price any damages for losses due to the nonconforming tender.

NOT- Accept the 800 pounds tendered, but notify the mill that the bakery will cancel the contract if the exact amount is not delivered on the following Monday.

64
Q

Jurisdictions that allow recovery for the loss of the chance of survival have created an exception to the traditional common law rules for establishing cause in fact. Under the traditional rules, the wife would be required to prove that reasonable action on the part of the hospital (presumably a correct diagnosis) would, more likely than not, have led to the man’s survival. Here, the wife cannot establish that the chances of the man’s survival would have been greater than 50% even if he had been given appropriate medical care. A jurisdiction that allows recovery for loss of the chance of survival, however, would allow the wife to recover for the reduction in her husband’s chance of surviving that was caused by the failure to properly diagnose.

A

A man was admitted to a hospital after complaining of persistent severe headaches. While he was there, hospital staff failed to diagnose his condition, and he was discharged. Two days later, the man died of a massive brain hemorrhage due to a congenital defect in an artery. The man’s wife has brought a wrongful death action against the hospital. The wife offers expert testimony that the man would have had a “reasonable chance” (not greater than 50%) of surviving the hemorrhage if he had been given appropriate medical care at the hospital.

In what type of jurisdiction would the wife’s suit most likely be successful?

  • jurisdiction that applies traditional common law rules concerning
  • A jurisdiction that allows recovery based on strict liability
  • A jurisdiction that allows recovery for the loss of the chance of survival.
  • A jurisdiction that recognizes loss of spousal consortium.
65
Q

$50,000 represents the contractor’s expectation measure of recovery and gives the contractor the benefit of the bargain—this amount would place the contractor in the position he would have been in but for the breach. It is also the greatest amount the contractor is able to recover. The general expectation formula permits the contractor to recover $50,000 and can be computed as follows: General expectation formula = loss in value + other loss – cost avoided – loss avoided: LIV = the difference between the performance the nonbreaching party should have received under the contract and what was actually received, if anything, in this case, $300,000 less $0 OL = consequential and incidental damages, if any, in this case, $0 CA = the additional costs the nonbreaching party can avoid by rightfully discontinuing performance under the contract as a result of the other party’s breach, in this case, $290,000 less $45,000 LA = the beneficial effects of the breach due to the nonbreaching party’s ability to salvage or reallocate resources that otherwise would have been devoted to performing under the contract, in this case, $5,000 So $300,000 + $0 – $245,000 – $5,000 = $50,000.

A

A homeowner and a contractor entered into a contract for the construction of a home for $300,000. The contractor was to earn a profit of $10,000 for the job. After the contractor had spent $45,000 on labor and materials, including $5,000 on oak flooring not yet installed, the homeowner informed the contractor that the homeowner had lost his job and could not pay for any services. The homeowner told the contractor to stop working immediately. The reasonable market value of the labor and materials provided by the contractor at that point, including the oak flooring, was $40,000. The contractor used the oak flooring on another job.

In an action by the contractor against the homeowner for damages, which of the following would be the largest amount of damages recoverable by the contractor?

  • $40,000, the reasonable value of the services the contractor had provided.
  • $40,000, the contractor’s construction costs.
  • $50,000, the contractor’s construction costs of $45,000 plus the $10,000 profit minus the $5,000 saved by reusing the oak flooring on another job.
  • $55,000, the contractor’s construction costs of $45,000 plus the $10,000 profit.
66
Q

Interrogatories may be directed to a corporate defendant, such as the manufacturer, which must designate an agent or officer to provide responses. Fed. R. Civ. P. 33(b)(1)(B).

A

A plaintiff brought a diversity action in federal court against a corporate manufacturer of revolving doors, alleging that a design flaw in the manufacturer’s doors had led the plaintiff to slip and fall, injuring herself, in a shopping center. After the plaintiff’s fall, the manufacturer modified the design of its doors and fired the employee responsible for the original design.

How, if at all, may the plaintiff’s attorney direct interrogatories to the manufacturer?

  • interrogatories may not be directed to a corporate defendant such as the manufacturer.
  • Interrogatories may only be directed to the manufacturer’s current employees, if any, who may have knowledge of the design process at issue.
  • Interrogatories may only be directed to an officer or agent of the manufacturer designated by the court.
  • Interrogatories may be directed to the manufacturer, and the manufacturer must designate an officer or agent to respond.
67
Q

The first bank had priority. The second bank was a necessary party to the foreclosure proceeding and was given notice of the sale. When the second bank failed to appear at the foreclosure proceeding, or to take any other action, the buyer at the sale received the title the farmer had at the time the mortgage was given to the first bank, which was a title free of any other mortgage liens. The buyer and the farmer did not act in collusion, so there could be no claim of fraud when the farmer reacquired her original interest in the farm.

A

A farmer borrowed $100,000 from a bank and gave the bank a promissory note secured by a mortgage on the farm that she owned. The bank promptly recorded the mortgage, which contained a due-on-sale provision. A few years later, the farmer borrowed $5,000 from a second bank and gave that bank a promissory note secured by a mortgage on her farm. The bank promptly recorded the mortgage. Subsequently, the farmer defaulted on her obligation to the first bank, which then validly accelerated the debt and instituted nonjudicial foreclosure proceedings as permitted by the jurisdiction. The second bank received notice of the foreclosure sale but did not send a representative to the sale. At the foreclosure sale, a buyer who was not acting in collusion with the farmer outbid all other bidders and received a deed to the farm. Several months later, the original farmer repurchased her farm from the buyer, who executed a warranty deed transferring the farm to her. After the farmer promptly recorded that deed, the second bank commenced foreclosure proceedings on the farm. The farmer denied the validity of the second bank’s mortgage.

Does the second bank continue to have a valid mortgage on the farm?

  • Yes, because of the doctrine of estoppel by deed.
  • Yes, because the original owner reacquired title to the farm.
  • No, because the purchase at the foreclosure sale by the buyer under these facts eliminated the second bank’s junior mortgage lien.
  • No, because of the due-on-sale provision in the farmer’s mortgage to the first bank.
68
Q

The Restatement (Second) of Contracts § 45 provides that where an offer invites acceptance by performance, the offeree’s beginning of performance creates an option contract which precludes the offeror from revoking its offer. Because the customer had begun his wait before the poster was taken down and because he ended up standing in line for more than five minutes, the bank should be liable for the $25.

A

While waiting in line to open an account with a bank, a customer read a poster on the bank’s wall that said, “New Customers! $25 FOR 5 MINUTES. If you stand in line for more than five minutes, we will pay you $25! We like happy customers!” The customer started timing his wait and just as five minutes was about to pass, the bank manager tore the poster down and announced, “The $25 stand-in-line promotion is over.” The customer waited in line for 10 more minutes before being served.

Does the customer have a claim against the bank for $25?

  • No, because the bank withdrew its offer before the customer completed the requested performance.
  • No, because the bank’s statement was a nonbinding gift promise.
  • Yes, because the bank could not revoke its offer once the customer had commenced performance.
  • Yes, because the customer’s presence in line served as notice to the bank that he had accepted.
69
Q

A driver stopped at a red traffic light in his home state. A stranger opened the passenger door, got in, and pointed a gun at the driver. The stranger then directed the driver to keep driving. They drove several miles, crossed into a neighboring state, and drove several more miles. When they reached a remote location, the stranger ordered the driver to pull over. The stranger then robbed the driver of his wallet and cash, and ordered him out of the car. The stranger drove off in the driver’s car. The stranger is charged with kidnapping in the neighboring state, which has adopted the Model Penal Code.

Can the stranger properly be convicted of kidnapping in the neighboring state?

  • Yes, because the driver was transported under threat of force in the neighboring state.
  • Yes, because the driver in effect paid a ransom for his release.
  • No, because any kidnapping took place in the driver’s home state.
  • No, because the restraint was incidental to the robbery.
A

Yes, because the driver was transported under threat of force in the neighboring state.

_

The movement in the neighboring state would constitute sufficient asportation (carrying away) for a kidnapping conviction, and was more than incidental to the robbery.

70
Q

A consumer bought a kitchen blender from the manufacturer. Soon after the purchase, the consumer was using the blender in an appropriate way when the blender jar shattered, throwing a piece of glass into the consumer’s eye. The consumer brought an action against the manufacturer based solely on strict products liability. The consumer’s expert testified that the blender was defectively designed. However, because the blender jar had been destroyed in the accident, the expert could not determine whether the accident had been caused by the design defect or a manufacturing defect. The manufacturer’s expert testified that the blender was not defective.

If, at the conclusion of the evidence, both parties move for directed verdicts, how should the trial judge rule?

  • Direct a verdict for the manufacturer, because the consumer’s expert was unable to specify the nature of the defect.
  • Direct a verdict for the manufacturer, because the consumer’s action was brought solely on a strict liability theory.
  • Direct a verdict for the consumer, because the blender was new when the jar shattered, and thus was undeniably defective.
  • Deny both motions and send the case to the jury, because a jury reasonably could conclude that the harm was caused by a defect present in the product when it was sold.
A

Deny both motions and send the case to the jury, because a jury reasonably could conclude that the harm was caused by a defect present in the product when it was sold.

_

A jury could conclude that the blender was defective at the time of sale because the accident was the sort of accident that ordinarily occurs as a result of a defect and no other cause was identified, but the jury is not required to draw that conclusion.

71
Q

Congressional committees heard testimony from present and former holders of licenses issued by state vocational licensing boards. According to the testimony, the boards had unfairly manipulated their disciplinary proceedings in order to revoke the licenses of some license holders as a means of protecting favored licensees from competition. In response, Congress enacted a statute prescribing detailed procedural requirements for the disciplinary proceedings of all state vocational licensing boards. The statute requires the state boards to provide licensees with adequate notice and opportunity for an adjudicatory hearing in all disciplinary proceedings. The statute also prescribes membership criteria for state vocational licensing boards, designed to ensure that the boards are likely to be neutral.

Which of the following provides the best source of authority for this federal statute?

  • Section 5 of the Fourteenth Amendment.
  • The general welfare clause of Article I, Section 8.
  • The privileges and immunities clause of Article IV, Section 2.
  • The takings clause of the Fifth Amendment.
A

Section 5 of the Fourteenth Amendment.

_

Section 5 of the Fourteenth Amendment gives Congress the power to enforce the provisions of the Fourteenth Amendment by appropriate legislation. Congressional legislation is appropriate within the meaning of Section 5 if (1) it seeks to prevent or remedy actions by state or local governments that violate provisions of the Fourteenth Amendment, and (2) its requirements are congruent with and proportional to the Fourteenth Amendment violations it addresses. In this case, the legislation seeks to prevent actions by state agencies that violate the due process clause of the Fourteenth Amendment, and the requirements of the legislation appear to be proportional to and congruent with the Fourteenth Amendment violations Congress has sought to prevent.

_

The privileges and immunities clause of Article IV, Section 2 prohibits actions by states that improperly discriminate against the citizens of other states. The clause does not apply in this situation because this statute does not address discrimination by licensing boards against the citizens of other states.

72
Q

A plaintiff sued a defendant law firm for malpractice in a diversity action in federal court. The plaintiff alleged that an attorney who had worked as an independent contractor of the firm had mishandled a legal matter for the plaintiff. In her complaint, the plaintiff alleged that the firm was liable, even though the attorney was an independent contractor, because the plaintiff had reasonably believed the attorney was an employee of the firm based on the attorney’s “apparent authority.” The courts of the state providing the applicable law have not recognized this theory of “apparent authority,” although other jurisdictions have adopted it. The defendant moved to dismiss the plaintiff’s complaint for failure to state a claim upon which relief can be granted.

Is the court likely to grant the defendant’s motion to dismiss?

  • Yes, because the applicable state law does not recognize the “apparent authority” theory.
  • No, because other jurisdictions have adopted the “apparent authority” theory.
  • No, because the plaintiff’s reasonable belief that the attorney was an employee is a question of fact.
  • No, because the court must take the allegations in the complaint as true, and the plaintiff has alleged that the law firm is liable for the acts of the attorney.
A

Yes, because the applicable state law does not recognize the “apparent authority” theory.

_

A complaint fails to state a claim if it fails to allege facts that constitute a cause of action under applicable law. See Fed. R. Civ. P. 8(a), 12(b). Here, the facts provide that applicable law does not recognize the “apparent authority” theory. Accordingly, the court will reject the plaintiff’s theory as a recoverable claim under applicable law. Options C and D are necessarily incorrect for the same reasons. Option B is incorrect because the outcome does not turn on applicable law in other jurisdictions, even though the court may ask the highest state court of the controlling jurisdiction to certify the issue and decide whether to adopt the theory.

73
Q

By a valid written contract executed in March, a seller agreed to sell land to a buyer. The contract stated, “The parties agree that closing will occur on May 1 at 10 a.m.” There was no other reference to closing. The contract was silent as to quality of title. On April 27, the seller notified the buyer that she had discovered that the land was subject to a long-standing easement in favor of a corporation for a towpath for a canal, should the corporation ever want to build a canal. The buyer thought it so unlikely that a canal would ever be built that the closing should occur notwithstanding this easement. Therefore, the buyer notified the seller on April 28 that he would expect to close on May 1. When the seller later refused to close, the buyer sued for specific performance.

Will the buyer be likely to prevail?

  • No, because the easement renders the seller’s title unmarketable.
  • No, because rights of third parties are unresolved.
  • Yes, because the decision to terminate the contract for title not being marketable belongs only to the buyer.
  • Yes, because the seller did not give notice of the easement a reasonable time before the closing date.
A

Yes, because the decision to terminate the contract for title not being marketable belongs only to the buyer.

_

If a contract of sale is silent as to quality of title, the court will imply a marketable title, and an easement does affect the marketability of title. But while the seller has a duty to deliver a marketable title, the requirement of marketable title is for the benefit of the buyer. The buyer may waive the right to have a marketable title, which is what the buyer did in this case.

74
Q

An insurance company issued an insurance policy to a homeowner. The policy failed to contain certain coverage terms required by a state insurance statute. When the homeowner suffered a loss due to a theft that was within the policy’s terms, the insurance company refused to pay, claiming that the policy was unenforceable because it violated the statute.

Will the homeowner likely succeed in an action against the insurance company to recover for the loss?

  • No, because the insurance policy is not a divisible contract.
  • No, because the insurance policy violated the statute.
  • Yes, because the homeowner belongs to the class of persons intended to be protected by the statute.
  • Yes, because the insurance policy will be strictly construed against the insurance company as the drafter.
A

Yes, because the homeowner belongs to the class of persons intended to be protected by the statute.

_

A contract that violates a state statute may be declared unenforceable on grounds of public policy. Where, however, the contract violates a policy that was intended for the benefit of a contracting party seeking relief, the contract may be enforceable in order to avoid frustrating the policy behind the statute. Accordingly, public policy would not prevent enforcement of the contract by those within the class of persons, including the homeowner, that the statute was intended to protect.

75
Q

While driving his pickup truck with a friend riding in the open bed, the driver swerved, throwing his friend to the pavement. The friend sustained severe injuries. The friend had often ridden in the open bed of the truck, and on some of those occasions the driver had swerved to frighten his friend. The friend sued the driver to recover both compensatory damages for his injuries and punitive damages.

Which cause of action would NOT permit the friend to recover punitive damages?

  • Assault
  • Battery.
  • Negligence.
  • Recklessness.
A

Negligence.

_

Punitive damages are not available in ordinary negligence cases.

Punitive damages are available to victims of intentional torts.

76
Q

A collector bought from a gallery a painting correctly described in the parties’ signed contract as a “one-of-a-kind self-portrait” by a famous artist who had recently died. The contract price was $500,000 in cash, payable one month after a truck carrier delivered the painting to the collector. The painting was damaged in transit. The collector timely rejected it after inspection, immediately notified the gallery of the rejection, and told the gallery that the painting would be available for pickup. The gallery then sold the painting to a third party. It informed the collector that it would pick up the painting within a couple of weeks. Two weeks later, before the gallery picked up the painting, the collector sold and delivered the painting to an art admirer for $550,000 cash, after notifying the admirer about the damage. The collector sent no money to the gallery.

If the collector’s sale of the painting was NOT an acceptance of the goods, what is the maximum amount that the gallery is entitled to recover from the collector?

  • $550,000 (damages for conversion).
  • $500,000 (the collector-gallery contract price)
  • $50,000 (the excess of the market price over the contract price)
  • Only the allowance of lost profit to the gallery as a volume dealer.
A

$550,000 (damages for conversion).

_

The collector rightfully rejected the goods. However, the collector’s exercise of ownership by selling the painting, after his rejection, was wrongful against the gallery and constituted conversion. The remedy for conversion is the fair market value of the goods at the time of the conversion. The collector’s sale of the painting for $550,000 provides credible evidence of the painting’s fair market value at the time of the conversion. UCC §§ 2-601 cmt. 2, 1-103(b).

77
Q

Six years ago, a landlord and a tenant entered into a 10-year commercial lease of land. The written lease provided that if a public entity under the power of eminent domain condemned any part of the land but not all of it, the lease would terminate and the landlord would receive the entire condemnation award. Thereafter, the city condemned approximately two-thirds of the land. The tenant notified the city and the landlord that an independent appraisal of the value of the tenant’s possessory interest established that it substantially exceeded the tenant’s obligation under the lease and that the tenant was entitled to share the award. The appraisal was accurate.

In an appropriate action among the landlord, the tenant, and the city as to the right of the tenant to a portion of the condemnation award, for whom will the court likely find?

  • The landlord, because the condemnation superseded and canceled the lease.
  • The landlord, because the parties specifically agreed as to the consequences of a partial condemnation.
  • The tenant, because the landlord breached the landlord’s implied warranty of quiet enjoyment.
  • The tenant, because otherwise the landlord would be unjustly enriched.
A

The landlord, because the parties specifically agreed as to the consequences of a partial condemnation.

_

78
Q

A landowner mortgaged her land to a nationally chartered bank as security for a loan. The mortgage provided that the bank could, at its option, declare the entire loan due and payable if all or any part of the land, or an interest therein, was sold or transferred without the bank’s prior written consent. Subsequently, the landowner wanted to sell the land to a neighbor by an installment land contract, but the bank refused to consent. The neighbor’s credit was good, and all mortgage payments to the bank were fully current. The landowner and the neighbor consulted an attorney about their proposed transaction, their desire to complete it, and the bank’s refusal to consent.

What would the attorney’s best advice be?

  • Even if the landowner transfers to the neighbor by land contract, the bank may accelerate the debt and foreclose if the full amount is not paid.
  • The due-on-sale clause is void as an illegal restraint on alienation of the fee simple, so they may proceed.
  • By making the transfer in land contract form, the landowner will prevent enforcement of the due-on-sale clause if the mortgage payments are kept current.
  • The only effect of the due-on-sale clause is that the proposed transfer will automatically make the neighbor personally liable on the debt, whether or not the neighbor specifically agrees to assume it.
A

Even if the landowner transfers to the neighbor by land contract, the bank may accelerate the debt and foreclose if the full amount is not paid.

_

The mortgage contains a valid due-on-sale clause. If the landowner conveys the land without the prior consent of the bank, the bank may accelerate the mortgage debt. A sale by use of an installment land contract is a transfer of the land, which can trigger the due-on-sale clause.

79
Q

A man decided to steal a car he saw parked on a hill. After he got in and started the engine, the car began rolling down the hill. The man quickly discovered that the car’s brakes did not work. He crashed through the window of a store located at the bottom of the hill. The man was charged with larceny of the car and with the crime of malicious damage to property. At trial, the judge instructed the jury that if the jury found both that the man was guilty of larceny of the car and that the damage to the store was the result of that larceny, then it should also find him guilty of malicious damage to property. The man was convicted on both counts. On appeal, he argued that the conviction for malicious damage to property should be reversed because the instruction was not a correct statement of the law.

Should the man’s conviction be affirmed?

  • Yes, because his intent to steal the car provides the necessary mental element.
  • Yes, because he was committing a felony.
  • No, because the instruction wrongly described the necessary mental state.
  • No, because it would violate double jeopardy to convict the man of two crimes for a single act.
A

No, because the instruction wrongly described the necessary mental state.

_

The instruction was wrong, because to have acted knowingly with respect to the malicious damage charge, the man must have been practically certain that his conduct would damage the store.

80
Q

Twenty-five years ago, a man who owned a 45-acre tract of land conveyed 40 of the 45 acres to a developer by warranty deed. The man retained the rear five-acre portion of the land and continues to live there in a large farmhouse. The deed to the 40-acre tract was promptly recorded. It contained the following language: “It is a term and condition of this deed, which shall be a covenant running with the land and binding on all owners, their heirs and assigns, that no use shall be made of the 40-acre tract of land except for residential purposes.” Subsequently, the developer fully developed the 40-acre tract into a residential subdivision consisting of 40 lots with a single-family residence on each lot. Although there have been multiple transfers of ownership of each of the 40 lots within the subdivision, none of them included a reference to the quoted provision in the deed from the man to the developer, nor did any deed to a subdivision lot create any new covenants restricting use. Last year, a major new medical center was constructed adjacent to the subdivision. A doctor who owns a house in the subdivision wishes to relocate her medical office to her house. For the first time, the doctor learned of the restrictive covenant in the deed from the man to the developer. The applicable zoning ordinance permits the doctor’s intended use. The man, as owner of the five-acre tract, however, objects to the doctor’s proposed use of her property. There are no governing statutes other than the zoning code. The common law Rule Against Perpetuities is unmodified in the jurisdiction.

May the doctor convert her house in the subdivision into a medical office?

  • No, because the owners of lots in the subdivision own property benefitted by the original residential covenant and have the sole right to enforce it.
  • No, because the man owns property benefitted by the original restrictive covenant and has a right to enforce it.
  • Yes, because the original restrictive covenant violates the Rule Against Perpetuities.
  • Yes, because the zoning ordinance allows the doctor’s proposed use and preempts the restrictive covenant.
A

No, because the man owns property benefitted by the original restrictive covenant and has a right to enforce it.

_

The restrictive covenant created 25 years ago placed a burden (that the land must be kept residential) on the 40-acre tract of land and gave the right to enforce the covenant to the man who retained ownership of the benefitted five-acre tract of land. The man may enforce the covenant because he owns land benefitted by it.

81
Q

The statement is admissible as a present sense impression under Rule 803(1) of the Federal Rules of Evidence because it describes an event and was made while the witness was perceiving the event or immediately thereafter. But the recording is also admissible under Rule 803(5) of the Federal Rules of Evidence because it is a record concerning a matter about which the witness once had knowledge but now has insufficient recollection to testify fully and accurately; it was made by the witness when the matter was fresh in the witness’s memory; and the witness has sufficiently established that the record was accurate when made.

A

A pedestrian sued a driver for injuries suffered in a hit-and-run accident. At trial, the pedestrian called a witness who testified that he saw the accident and that as the car sped off he accurately dictated the license number into his properly operating pocket dictating machine. The witness has stated that he no longer remembers the number.

May the tape recording be played?

  • Yes, as a present sense impression only.
  • Yes, as a recorded recollection only.
  • Yes, as a present sense impression and as a recorded recollection.
  • No, because it is hearsay not within any exception.
82
Q

In a city, a number of armed bank robberies were committed near closing time by a masked man wearing a white hooded sweatshirt and blue sweatpants. Police saw a man wearing a white hooded sweatshirt and blue sweatpants pacing nervously outside one of the city’s banks just before it closed. The police stopped the man and frisked the outer layers of his clothing for weapons, but found none. They asked the man what he was doing outside the bank and pointed out that he was wearing clothing similar to clothing worn by the perpetrator of recent robberies. After pausing for several moments, the man confessed. The police had not provided him with any Miranda warnings. After being charged with the bank robberies, the man moved to suppress his confession. The parties agreed, and the court properly found, that the police had reasonable suspicion, but not probable cause, at all times before the man confessed.

Should the man’s motion to suppress be granted?

  • Yes, because the confession was the fruit of a Fourth Amendment violation, even though there was no Miranda violation.
  • Yes, because the confession was the fruit of a Miranda violation, even though there was no Fourth Amendment violation.
  • Yes, because the confession was the fruit of both a Fourth Amendment violation and a Miranda violation.
  • No, because there was neither a Fourth Amendment violation nor a Miranda violation.
A

No, because there was neither a Fourth Amendment violation nor a Miranda violation.

_

There was neither a Fourth Amendment violation nor a Miranda violation. There was no Fourth Amendment violation because the stop, frisk, and questioning were permissible, under Terry v. Ohio, 392 U.S. 1 (1968), based on reasonable suspicion. There was no Miranda violation because warnings are not required for Terry stops.

83
Q

At the trial of a contract dispute, the plaintiff has offered to testify to what she heard the defendant say in a private conversation between the two of them, which the plaintiff secretly recorded on an audiotape that she did not offer in evidence.

Is the plaintiff’s testimony admissible?

  • Yes, because the plaintiff has personal knowledge of the statement of a party-opponent.
  • Yes, because the original document rule does not apply to audiotapes.
  • No, because the statement must be proved by introduction of the audiotape itself.
  • No, because of the plaintiff’s deception, even if the recording was not illegal.
A

Yes, because the plaintiff has personal knowledge of the statement of a party-opponent.

_

What the defendant said to the plaintiff, even in a private conversation, is a statement of a party-opponent and is admissible under Rule 801(d)(2)(A) of the Federal Rules of Evidence. The plaintiff has personal knowledge of what the defendant said and can testify about it. The fact that the audiotape might be better evidence of what the defendant actually said makes no difference. The best evidence rule applies only when a witness testifies about the content of a writing or recording. Here the plaintiff would not be testifying about the content of the audiotape but rather about what she personally heard.

84
Q

A plaintiff sued a ladder manufacturer for injuries he suffered to his neck and back when a rung of the ladder on which he was standing gave way. When the plaintiff’s back and neck continued to be very sore more than two weeks after his fall, his treating physician had sent him to an orthopedist for an evaluation. Though the orthopedist did not treat the plaintiff, he diagnosed an acute cervical strain. At trial, the plaintiff has called the orthopedist to testify that, in response to the orthopedist’s inquiry about how the plaintiff had injured his back, the plaintiff had told him, “I was standing near the top of a 15-foot ladder when I abruptly fell, landing hard on my back, after which the ladder toppled onto my neck.”

Should the statement be admitted?

  • Yes, because the plaintiff is present and can be cross-examined about it.
  • Yes, because it was made for the purpose of medical diagnosis or treatment.
  • No, because it was not made to a treating physician.
  • No, because it relates to the inception or the cause of the injury rather than the plaintiff’s physical condition.
A

Yes, because it was made for the purpose of medical diagnosis or treatment.

_

This statement fits Rule 803(4) of the Federal Rules of Evidence as a statement made for the purpose of medical diagnosis. This rule allows not only statements made to treating physicians, but also statements made to other doctors for evaluation or diagnosis-including doctors consulted for diagnoses for purposes of litigation.

85
Q

The United States government demonstrated that terrorist attacks involving commercial airliners were perpetrated exclusively by individuals of one particular race. In response, Congress enacted a statute imposing stringent new airport and airline security measures only on individuals of that race seeking to board airplanes in the United States.

Which of the following provides the best ground for challenging the constitutionality of this statute?

  • The commerce clause of Article I, Section 8.
  • The due process clause of the Fifth Amendment.
  • The privileges and immunities clause of Article IV.
  • The privileges or immunities clause of the Fourteenth Amendment.
A

_

  • The due process clause of the Fifth Amendment.

In Bolling v. Sharpe, 347 U.S. 497 (1954), the U.S. Supreme Court held that the equal protection principles of the Fourteenth Amendment apply to actions of the federal government through the due process clause of the Fifth Amendment. The new security measures presumptively violate equal protection because they contain a racial classification: they apply only to individuals of one particular race. A court therefore would uphold the new security measures only if the government could prove that they are necessary to serve a compelling public interest, a standard that the government typically cannot meet.

86
Q

On March 1, a homeowner contacted a builder about constructing an addition to the homeowner’s house. The builder orally offered to perform the work for $200,000 if his pending bid on another project was rejected. The homeowner accepted the builder’s terms, and the builder then prepared a written contract that both parties signed. The contract did not refer to the builder’s pending bid. One week later, upon learning that his pending bid on the other project had been accepted, the builder refused to perform any work for the homeowner.

Can the homeowner recover for the builder’s nonperformance?

  • No, because efficiency principles justify the builder’s services being directed to a higher-value use.
  • No, because the builder’s duty to perform was subject to a condition.
  • Yes, because the builder’s attempt to condition his duty to perform rendered the contract illusory.
  • Yes, because the parol evidence rule would bar the builder from presenting evidence of oral understandings not included in the final writing.
A

No, because the builder’s duty to perform was subject to a condition.

_

The condition exception to the parol evidence rule permits the admission of extrinsic evidence to establish an oral condition to the parties’ performance under the contract, which, because it was not satisfied, discharged the builder’s duty to perform.

87
Q

Residents of a city complained that brightly colored signs detracted from the character of the city’s historic district and distracted motorists trying to navigate its narrow streets. In response, the city council enacted an ordinance requiring any “sign or visual display” visible on the streets of the historic district to be black and white and to be no more than four feet long or wide. A political party wanted to hang a six-foot-long red, white, and blue political banner in front of a building in the historic district. The party filed suit to challenge the constitutionality of the sign ordinance as applied to the display of its banner.

Which of the following would be the most useful argument for the political party?

  • The ordinance is not the least restrictive means of promoting a compelling government interest.
  • The ordinance is not narrowly tailored to further an important government interest, nor does it leave open alternative channels of communication.
  • The ordinance imposes a prior restraint on political expression.
  • The ordinance effectively favors some categories of speech over others.
A

The ordinance is not narrowly tailored to further an important government interest, nor does it leave open alternative channels of communication.

_

Because the ordinance is a content-neutral restriction of expression, it must satisfy intermediate scrutiny, which requires the city to prove that the ordinance is narrowly tailored to further an important government interest and that it leaves open alternative channels of communication.

88
Q

A foreign diplomat discovered that a small person could enter a particular jewelry store by crawling through an air vent. The diplomat befriended a woman he met in a bar who he believed was small enough to crawl through the air vent. Without telling her that he was a diplomat, he explained how she could get into the jewelry store. She agreed to help him burglarize the store. Someone overheard their conversation and reported it to the police. Shortly thereafter, the police arrested the diplomat and the woman. Both were charged with conspiracy to commit burglary. Before trial, the diplomat moved to dismiss the charge against him on the ground that he was entitled to diplomatic immunity. The court granted his motion. The woman then moved to dismiss the conspiracy charge against her. The jurisdiction has adopted the Model Penal Code version of conspiracy.

Should the court grant the woman’s motion to dismiss the conspiracy charge against her?

  • No, because the diplomat’s defense does not negate any element of the crime.
  • No, because the woman was not aware of the diplomat’s status
  • Yes, because a conspiracy requires two guilty participants
  • Yes, because but for the diplomat’s conduct, no conspiracy would have occurred.
A

No, because the diplomat’s defense does not negate any element of the crime.

_

Model Penal Code section 5.03(1) adopts a unilateral interpretation of conspiracy, defining conspiracy as requiring agreement by the defendant but not by two or more persons. In addition, many jurisdictions that require a bilateral conspiracy still allow conviction if one of the co-conspirators agreed to the crime but cannot be convicted based on lack of capacity or some other defense personal to the co-conspirator.

89
Q

A plaintiff sued his insurance company for the full loss of his banquet hall by fire. The insurance company defended under a provision of the policy limiting liability to 50 percent if “flammable materials not essential to the operation of the business [were] stored on the premises and cause[d] a fire.” The insurance company called the keeper of the city fire inspection records to identify a report prepared and filed by the fire marshal as required by law, indicating that shortly before the fire, the fire marshal had cited the plaintiff for storing gasoline at the banquet hall.

Is the report admissible?

  • No, because it is hearsay not within any exception
  • No, because the proceeding is civil, rather than criminal.
  • Yes, as a public record describing matters observed as to which there was a duty to report.
  • Yes, as a record of regularly conducted activity, provided that the fire marshal is unavailable.
A

Yes, as a public record describing matters observed as to which there was a duty to report.

_

The report is admissible as a public record under Rule 803(8) of the Federal Rules of Evidence. The facts of the problem indicate that the fire marshal had a legal duty to report. The fact that the fire marshal issued the citation indicates that he observed gasoline being stored in the banquet hall.

90
Q

A rectangular parcel of undeveloped land contained three acres and had 150 feet of frontage on a public street. The applicable zoning ordinance required that a buildable lot contain at least two acres and have frontage of not less than 100 feet on a public street. A brother and sister owned the land as tenants in common, the brother owning a one-third interest and the sister owning a two-thirds interest. Neither of them owned any other real property. The sister brought an appropriate action to partition the land and proposed that a two-acre rectangular lot with 100 feet of frontage be set off to her and that a one-acre rectangular lot with 50 feet of frontage be set off to the brother. The brother’s defense included a demand that the land be sold and its proceeds be divided one-third to the brother and two-thirds to the sister.

Who will prevail?

  • The brother, because partition by sale is the preferred remedy, unless a fair price is not the likely result of a sale.
  • The brother, because the zoning ordinance makes it impossible to divide the land fairly.
  • The sister, because partition by sale is not appropriate if the subject property can be physically divided.
  • The sister, because the ratio of the two lots that would result from her proposal conforms exactly to the ownership ratio.
A

The brother, because the zoning ordinance makes it impossible to divide the land fairly.

_

A tenant in common may bring an action to partition the property. Partition in kind, in which there is a physical division of the common property, is preferred; however, a partition by sale is allowed when a fair and equitable physical division of the property is impossible. The applicable zoning ordinance requires a frontage of 100 feet on a public street in order to build. It would not be fair or equitable to convey only 50 feet of frontage to the brother, who then could not build on his lot.

91
Q

After his home internet connection caught fire, a man sued his internet provider in Delaware federal court under the federal Telecommunications Act of 1996. The man was a resident of Delaware. The internet provider was incorporated in Maryland, but its principal place of business was in Delaware and all of its hardware was manufactured in New Jersey.

What substantive law should the court use in making its decision?

  • The applicable law of Delaware, because the federal court sits in Delaware.
  • The applicable law of Delaware, because the defendant’s principal place of business is in Delaware.
  • The applicable law of Maryland, because the defendant is incorporated in Maryland.
  • The applicable federal law, because the man’s claim is based on the Telecommunications Act.
A

The applicable federal law, because the man’s claim is based on the Telecommunications Act.

_

28 U.S.C. § 1652 states that federal courts should apply substantive federal law when dealing with a claim based on federal law. Here, since the man is suing under the federal Telecommunications Act, the court should apply substantive federal law.

92
Q

The marijuana plants were in plain view of the neighbors, and the woman has no standing to complain of any police trespass on the neighbors’ property.

A

The police suspected a woman of growing marijuana in her private residence. Narcotics officers went to her neighborhood in the middle of the night. Nothing unlawful could be seen from the street, so the officers walked into her neighbors’ yard and looked in through the woman’s kitchen window, which had neither drapes nor shades. The officers observed what appeared to be marijuana plants being cultivated under grow lights in the kitchen. Using this information, the officers obtained a search warrant. The execution of the warrant netted numerous marijuana plants. The woman was charged with possession of marijuana. She has moved to suppress the marijuana plants recovered when the warrant was executed, claiming that the evidence supporting the warrant was obtained through a search that violated the Fourth Amendment.

Should the marijuana plants be suppressed?

No, because the woman could have no reasonable expectation of privacy concerning activities that she exposed to the view of her neighbors.

93
Q

When a party effectively delegates his or her duties under an existing contract to a third party, such delegation does not discharge the delegator’s liability under the contract. Here, although the original carpenter properly delegated his duty to perform to an equally qualified second carpenter, the delegation of this duty did not discharge the original carpenter’s liability under the contract. Therefore, the original carpenter may be held liable for the second carpenter’s nonperformance and may be sued by the homeowner. The homeowner cannot sue the second carpenter who was delegated the duty to perform, because the second carpenter was not a party to the contract and cannot be held liable under it. Note that depending on the details of the delegation agreement, the original carpenter may be able to sue the second carpenter.

A

A carpenter contracted to construct a gazebo for a homeowner, who intended to use the gazebo for his daughter’s upcoming wedding. A week before the carpenter was scheduled to begin construction, he realized that he had overcommitted himself and was unlikely to be able to complete the gazebo on time for the wedding. The carpenter delegated his duty to perform to an equally qualified second carpenter, promising to pay the second carpenter the same amount that the homeowner had promised to pay for the work. Neither carpenter did any work on the gazebo contract.

Against whom is the homeowner likely to recover damages for breach of contract?

Against the original carpenter only.

94
Q

The mortgage contains a valid due-on-sale clause. If the landowner conveys the land without the prior consent of the bank, the bank may accelerate the mortgage debt. A sale by use of an installment land contract is a transfer of the land, which can trigger the due-on-sale clause.

A

A landowner mortgaged her land to a nationally chartered bank as security for a loan. The mortgage provided that the bank could, at its option, declare the entire loan due and payable if all or any part of the land, or an interest therein, was sold or transferred without the bank’s prior written consent. Subsequently, the landowner wanted to sell the land to a neighbor by an installment land contract, but the bank refused to consent. The neighbor’s credit was good, and all mortgage payments to the bank were fully current. The landowner and the neighbor consulted an attorney about their proposed transaction, their desire to complete it, and the bank’s refusal to consent.

What would the attorney’s best advice be?

Even if the landowner transfers to the neighbor by land contract, the bank may accelerate the debt and foreclose if the full amount is not paid.

95
Q

The case arguably is not ripe for adjudication because the agency’s inspection does not itself pose any risk of harm to residents of the community. The residents face a risk of harm only if the agency selects their community as a site for a landfill, but on these facts it is unclear whether or when the community would be selected.

A

In response to the need for additional toxic waste landfills in a state, the state’s legislature enacted a law authorizing a state agency to establish five new state-owned and state-operated toxic waste landfills. The law provided that the agency would decide the locations and sizes of the landfills after an investigation of all potential sites and a determination that the particular sites chosen would not endanger public health and would be consistent with the public welfare. A community in the state was scheduled for inspection by the agency as a potential toxic waste landfill site. Because the community’s residents obtained most of their drinking water from an aquifer that ran under the entire community, a citizens’ group, made up of residents of that community, sued the appropriate officials of the agency in federal court. The group sought a declaratory judgment that selecting their community as the site of a toxic waste landfill would be unconstitutional and an injunction preventing the agency from doing so. The agency officials moved to dismiss.

Which of the following is the most appropriate basis for the court to dismiss this suit?

  • The case is not ripe for a decision on the merits.
  • The Eleventh Amendment bars suits of this kind in the federal courts.
  • The interest of the state in obtaining suitable sites for toxic waste landfills is sufficiently compelling to justify the selection of the community as a location for such a facility.
  • The case presents a nonjusticiable political question.
96
Q

The statement is admissible as a present sense impression under Rule 803(1) of the Federal Rules of Evidence because it describes an event and was made while the witness was perceiving the event or immediately thereafter. But the recording is also admissible under Rule 803(5) of the Federal Rules of Evidence because it is a record concerning a matter about which the witness once had knowledge but now has insufficient recollection to testify fully and accurately; it was made by the witness when the matter was fresh in the witness’s memory; and the witness has sufficiently established that the record was accurate when made.

A

A pedestrian sued a driver for injuries suffered in a hit-and-run accident. At trial, the pedestrian called a witness who testified that he saw the accident and that as the car sped off he accurately dictated the license number into his properly operating pocket dictating machine. The witness has stated that he no longer remembers the number.

May the tape recording be played?

  • Yes, as a present sense impression only.
  • Yes, as a recorded recollection only.
  • Yes, as a present sense impression and as a recorded recollection.
97
Q

The man could properly be convicted of murder, even though he lacked specific intent to kill, because his conduct created such a high risk of death and was so devoid of social utility that he could be found to have acted with a depraved heart.

A

A man became angry after he was unexpectedly laid off from his longtime job as a factory worker. The next day, he returned to the factory floor and indiscriminately fired shotgun rounds into the air. The man later testified, without contradiction, that he had not intended to kill anyone but had simply sought to exact revenge on the factory’s owners by shutting down operations for the day. Unfortunately, one of the bullets ricocheted off the wall and killed the man’s best friend. The crimes below are listed in descending order of seriousness.

On these facts, what is the most serious offense of which the man properly could be convicted?

  • Murder.
  • Voluntary manslaughter
  • Involuntary manslaughter.
  • Assault.
98
Q

The characterization of employment as “permanent” creates an employment-at-will relationship. In an employment-at-will relationship, either party can terminate the agreement at any time, without the termination being considered a breach (unless the termination violates an important public policy (which is not the case here). Accordingly, the chef is not liable for breach.

A

A bakery offered a chef a permanent full-time job as a pastry chef at a salary of $3,000 per month. The chef agreed to take the position and to begin work in two weeks. In her employment application, the chef had indicated that she was seeking a permanent job. One week after the chef was hired by the bakery, a hotel offered the chef a position as a restaurant manager at a salary of $3,500 a month. The chef accepted and promptly notified the bakery that she would not report for work at the bakery.

Is the bakery likely to prevail in a lawsuit against the chef for breach of contract?

  • No, because a contract for permanent employment would be interpreted to mean that the chef could leave at any time.
  • No, because the position the chef took with the hotel was not substantially comparable to the one she had agreed to take with the bakery.
  • Yes, because the chef’s acceptance of a permanent position meant that she agreed to leave the bakery only after a reasonable time.
  • Yes, because the chef’s failure to give the bakery a chance to match the salary offered by the hotel breached the implied right of first refusal.
99
Q

A deed must be delivered to be valid. Delivery is a question of intent. The words of the landowner included “This is yours,” showing the necessary intent to strip himself of dominion and control and to immediately transfer the title. In addition, handing the deed to the friend raises a rebuttable presumption of delivery. Recording the deed is not required to transfer title, and thus the request not to record the document until later is irrelevant so long as delivery was made.

A

A landowner executed an instrument in the proper form of a deed, purporting to convey his land to a friend. The landowner handed the instrument to the friend, saying, “This is yours, but please do not record it until after I am dead. Otherwise, it will cause me no end of trouble with my relatives.” Two days later, the landowner asked the friend to return the deed to him because he had decided that he should devise the land to the friend by will rather than by deed. The friend said that he would destroy the deed and a day or so later falsely told the landowner that the deed had been destroyed. Six months ago, the landowner, who had never executed a will, died intestate, survived by a daughter as his sole heir. The day after the landowner’s death, the friend recorded the deed from him. As soon as the daughter discovered this recording and the friend’s claim to the land, she brought an appropriate action against the friend to quiet title to the land.

For whom should the court hold?

  • The daughter, because the death of the landowner deprived the subsequent recording of any effect.
  • The daughter, because the friend was dishonest in reporting that he had destroyed the deed.
  • The friend, because the deed was delivered to him
  • The friend, because the deed was recorded by him.
100
Q

A shopper was riding on an escalator in a department store when the escalator stopped abruptly. The shopper lost her balance and fell down the escalator steps, sustaining injuries. Although the escalator had been regularly maintained by an independent contractor, the store’s obligation to provide safe conditions for its invitees was nondelegable. The shopper has brought an action against the store for damages, and the above facts are the only facts in evidence. The store has moved for a directed verdict.

Should the court grant the motion?

  • No, because the finder of fact could infer that the escalator malfunction was due to negligence.
  • No, because the store is strictly liable for the shopper’s injuries.
  • Yes, because an independent contractor maintained the escalator.
  • Yes, because the shopper has not produced evidence of negligence.
A

_

  • No, because the finder of fact could infer that the escalator malfunction was due to negligence.
101
Q

For 22 years, the land records have shown a man as the owner of an 80-acre farm. The man has never physically occupied the land. Nineteen years ago, a woman entered the farm. The character and duration of the woman’s possession of the farm caused her to become the owner of the farm under the adverse possession law of the jurisdiction. Three years ago, when the woman was not present, a neighbor took over possession of the farm. The neighbor repaired fences, put up “no trespassing” signs, and did some plowing. When the woman returned, she found the neighbor in possession of the farm. The neighbor vigorously rejected the woman’s claimed right to possession and threatened force. The woman withdrew. The woman then went to the man and told him of the history of activity on the farm. The woman orally told the man that she had been wrong to try to take his farm. She expressly waived any claim she had to the land. The man thanked her. Last month, unsure of the effect of her conversation with the man, the woman executed a deed purporting to convey the farm to her son. The son promptly recorded the deed. The period of time to acquire title by adverse possession in the jurisdiction is 10 years.

Who now owns the farm?

AThe man, because the woman’s later words and actions released title to the man.

BThe neighbor, because the neighbor succeeded to the woman’s adverse possession title by privity of possession.

CThe son, because he succeeded to the woman’s adverse possession title by privity of conveyance.

DThe woman, because she must bring a quiet title action to establish her title to the farm before she can convey the farm to her son.

A

The son, because he succeeded to the woman’s adverse possession title by privity of conveyance.

-

The woman acquired title to the farm by adverse possession. The woman’s title was an original title and did not derive from the man’s title. The statute of frauds requires that the conveyance of the farm be in writing. Therefore, the woman’s oral statement was insufficient to release the title to the man, and the woman validly conveyed the farm to her son.

102
Q

A woman’s car was set on fire by vandals. When she submitted a claim of loss for the car to her insurance company, the insurance company refused to pay, asserting that the woman’s policy had lapsed due to the nonpayment of her premium. The woman sued the insurance company for breach of contract. At trial, the woman testified that she had, in a timely manner, placed a stamped, properly addressed envelope containing the premium payment in the outgoing mail bin at her office. The woman’s secretary then testified that every afternoon at closing time he takes all outgoing mail in the bin to the post office. The insurance company later called its mail clerk to testify that he opens all incoming mail and that he did not receive the woman’s premium payment. The woman and the insurance company have both moved for a directed verdict.

For which party, if either, should the court direct a verdict?

  • For neither the woman nor the insurance company, because under these circumstances the jury is responsible for determining whether the insurance company received the payment.
  • For the woman, because there is a presumption that an envelope properly addressed and stamped was received by the addressee.
  • For the insurance company, because neither the woman nor her secretary has any personal knowledge that the envelope was delivered to the post office.
  • For the insurance company, because the mail clerk’s direct testimony negates the woman’s circumstantial evidence.
A

For neither the woman nor the insurance company, because under these circumstances the jury is responsible for determining whether the insurance company received the payment.

-

The woman has presented sufficient evidence to trigger the presumption that her payment was received. The insurance company has presented sufficient evidence to rebut that presumption. Consequently, the presumption is taken out of the case and it is up to the fact finder to determine whether the insurance company received the payment. Therefore, it would be error to grant a directed verdict for either the woman or the insurance company.

103
Q

In a diversity action for negligence filed in federal district court, the defendant refused to waive service. The plaintiff served an attorney who had previously represented the defendant in his divorce, at the attorney’s office. The attorney forwarded a copy of the pleadings to the defendant. The defendant thereafter moved to dismiss based on insufficient service.

If the court denies the defendant’s motion to dismiss, what is the likely reason?

  • Insufficient service may only be raised where the defendant did not receive actual notice of the plaintiff’s lawsuit.
  • Insufficient service may only be raised in the defendant’s answer.
  • An attorney may always receive service on behalf of past and present clients.
  • Courts tend to apply service of process rules liberally where the defendant received actual notice of the plaintiff’s lawsuit.
A

Courts tend to apply service of process rules liberally where the defendant received actual notice of the plaintiff’s lawsuit.

-

Under the Federal Rules of Civil Procedure (FRCP), service can be made upon an agent authorized by appointment to receive service of process. See FRCP 4(e)(2). Often a person’s attorney is contractually authorized to receive service on his or her behalf. Here, however, it is unclear whether the attorney was so authorized or was still so authorized following the completion of the defendant’s divorce proceedings. However, a court will typically construe the service rules liberally to avoid deciding a case on a technicality where the defendant did actually receive notice of the plaintiff’s lawsuit. See Nowell v. Nowell, 384 F.2d 951 (5th Cir. 1967).

104
Q

On June 15, a teacher accepted a contract for a one-year position teaching math at a public high school at a salary of $50,000, starting in September. On June 22, the school informed the teacher that, due to a change in its planned math curriculum, it no longer needed a full-time math teacher. The school offered instead to employ the teacher as a part-time academic counselor at a salary of $20,000, starting in September. The teacher refused the school’s offer. On June 29, the teacher was offered a one-year position to teach math at a nearby private academy for $47,000, starting in September. The teacher, however, decided to spend the year completing work on a graduate degree in mathematics and declined the academy’s offer.

If the teacher sues the school for breach of contract, what is her most likely recovery?

  • $3,000, the full contract amount less the amount the teacher could have earned in the teaching position at the academy.
  • $30,000, the full contract amount less the amount the teacher could have earned in the counselor position offered by the school.
  • $50,000, the full contract amount.
  • Nothing, because the school notified the teacher of its decision before the teacher had acted in substantial reliance on the contract.
A

The niece has a defeasible fee simple determinable and the daughter has an executory interest.

_

The niece has a defeasible fee simple because of the limitation placed on the estate by the words until my niece’s daughter marries. If the niece’s daughter marries, the estate in the niece will end automatically and will pass to the holder of the future interest (the niece’s daughter). The future interest given to the daughter, a grantee, is an executory interest. The executory interest in this case does not violate the common law Rule Against Perpetuities, because it will be known within the lifetime of the validating lives—the niece and the niece’s daughter—whether the condition of marriage has occurred.

105
Q

A driver sued her insurance company on an accident insurance policy covering personal injuries to the driver. The insurance company defended on the ground that the driver’s injuries were intentionally self-inflicted and therefore excluded from the policy’s coverage. The driver testified at trial that she had inflicted the injuries, as her negligence had caused the crash in which she was injured, but that she had not done so intentionally. She then called as a witness her treating psychiatrist to give his opinion that the driver had been mentally unbalanced, but not self-destructive, at the time of the crash.

Should the court admit the witness’s opinion?

ANo, because it is a statement about the driver’s credibility.

BYes, because it is a helpful opinion by a qualified expert.

CNo, because it is an opinion about a mental state that constitutes an element of the defense.

DNo, because the witness did not first state the basis for his opinion.

A

Yes, because it is a helpful opinion by a qualified expert.

_

The witness’s opinion helps the jury understand a relevant mental state. The standard for qualification of an expert is not high; a psychiatrist is qualified to testify to a person’s mental state.

106
Q

A contractor purchased a year’s supply of paint from an online vendor. When the order arrived, the contractor discovered that the paints were all dried. The contractor emailed the vendor, demanding a refund, but the vendor never responded. The contractor then filed a breach-of-contract action against the vendor in federal court, where diversity jurisdiction was proper. The vendor was properly served; however, it failed to answer the complaint. The clerk of the court entered a default on the court’s docket in the contractor’s favor.

Upon receiving notice of the entry of default, what should the contractor do next in order to recover against the vendor?

  • Execute upon the default by seizing any assets the vendor may have in its headquarters.
  • Execute upon the default by moving the federal court to issue a writ of execution against the vendor.
  • Execute upon the default by seizing any assets the vendor may have in the state where the complaint was filed.
  • Request that the court enter judgment against the vendor based on the vendor’s default.
A

Request that the court enter judgment against the vendor based on the vendor’s default.

_

Rule 55 of the Federal Rules of Civil Procedure (FRCP) sets forth the two steps necessary for defaulting an adversary in federal court: (1) a party must first request an entry of default on the docket from the clerk of the court by providing sufficient documents to show that the opposing party has defaulted on its obligation to respond properly to a pleading against it; (2) the party must then seek an entry of judgment against the opposing party based on the default.

Although the entry of default admits the well-pleaded allegations of a plaintiff’s complaint, it is not until the entry of a default judgment that an award is enforceable. Here, the clerk entered the vendor’s default on the docket. However, the contractor cannot attempt to collect any damages awarded until the court enters judgment on the default (either through the judge or through the clerk if the contractor’s damages are certain).

107
Q

A producer contracted to pay an inexperienced performer a specified salary to act in a small role in a play the producer was taking on a six-week road tour. The contract was for the duration of the tour. On the third day of the tour, the performer was hospitalized with a stomach disorder. The producer replaced her in the cast with an experienced actor. One week later, the performer recovered, but the producer refused to allow her to resume her original role for the remainder of the tour.

In an action by the performer against the producer for breach of contract, which of the following, if proved, would be the producer’s best defense?

  • The actor, by general acclaim, was much better in the role than the performer had been.
  • The actor was the only replacement the producer could find, and the actor would accept nothing less than a contract for the remainder of the six-week tour.
  • The producer offered to employ the performer as the actor’s understudy for the remainder of the six-week tour at the performer’s original salary, but the performer declined.
  • Both the producer and the performer knew that a year earlier the performer had been incapacitated for a short period of time by the same kind of stomach disorder.
A

The actor was the only replacement the producer could find, and the actor would accept nothing less than a contract for the remainder of the six-week tour.

_

After the performer became ill, the temporary impracticability doctrine excused the performer’s contractual obligation and also gave the producer the right to suspend his performance obligation during the period that the performer’s illness prevented her from acting. The critical issue here is whether the producer also had the right to cancel the contract. Circumstances that would give the producer the right to cancel the contract include the degree of uncertainty relating to the nature and duration of the performer’s illness and the extent to which a delay in making substitute arrangements would have prevented the producer from continuing the tour. The unwillingness of the actor, the only replacement available, to take a contract for less than the remainder of the six-week tour and the uncertainty surrounding when the performer might return to work would have discharged the producer’s performance obligations and justified his cancellation of the contract with the performer.

108
Q

A number of psychotherapists routinely send mailings to victims of car accidents informing the victims of the possibility of developing post-traumatic stress disorder (PTSD) as a result of the accidents, and offering psychotherapy services. Although PTSD is a possible result of a car accident, it is not common. Many accident victims in a particular state who received the mailings complained that the mailings were disturbing and were an invasion of their privacy. These victims also reported that as a result of the mailings, their regard for psychotherapists and for psychotherapy as a form of treatment had diminished. In response, the state enacted a law prohibiting any licensed psychotherapist from sending mailings that raised the concern of PTSD to any car accident victim in the state until 30 days after the accident. The state justified the law as an effort to address the victims’ complaints as well as to protect the reputation of psychotherapy as a form of treatment.

Is this law constitutional?

  • No, because the law singles out one type of message for prohibition while allowing others.
  • No, because the mailings provide information to consumers.
  • Yes, because mailings suggesting the possibility of developing PTSD as the result of an accident are misleading.
  • Yes, because the law protects the privacy of accident victims and the public regard for psychotherapy without being substantially more restrictive than necessary.
A

Yes, because the law protects the privacy of accident victims and the public regard for psychotherapy without being substantially more restrictive than necessary.

_

The law is constitutional, because it satisfies the First Amendment standards for government restrictions on commercial speech. The mailings contain commercial speech, because they advertise services provided by the psychotherapists. A restriction on commercial speech is subject to a form of intermediate judicial scrutiny, requiring the government to show that the restriction directly advances an important government interest and that the restriction is not substantially more extensive than necessary to protect that interest. The law here satisfies that standard; the 30-day waiting period for the psychotherapists’ mailings narrowly serves the government’s substantial interests in protecting both the privacy of accident victims and the public regard for psychotherapy.

109
Q
A
110
Q

A produce distributor contracted to provide a grocer with eight crates of lettuce at the distributor’s listed price. The distributor’s shipping clerk mistakenly shipped only seven crates to the grocer. The grocer accepted delivery of the seven crates but immediately notified the distributor that the delivery did not conform to the contract. The distributor’s listed price for seven crates of lettuce was 7/8 of its listed price for eight crates. The distributor shipped no more lettuce to the grocer, and the grocer has not yet paid for any of the lettuce.

How much, if anything, is the distributor entitled to collect from the grocer?

ANothing, because the tender of all eight crates was a condition precedent to the grocer’s duty to pay.

BThe reasonable value of the seven crates of lettuce, minus the grocer’s damages, if any, for the distributor’s failure to deliver the full order.

CThe listed price for the seven crates of lettuce, minus the grocer’s damages, if any, for the distributor’s failure to deliver the full order.

DThe listed price for the seven crates of lettuce.

A
111
Q

A plaintiff brought a diversity action against a defendant in federal court, seeking to recover damages for negligence. The plaintiff’s claim turned in part on a novel interpretation of a state statute, which the plaintiff argued established the defendant’s duty to the plaintiff as well as the applicable standard of care in the case. The defendant argued that the statute created no duty and moved to dismiss. The district court judge denied the defendant’s motion, and the case proceeded to trial. The jury found that the defendant had been negligent, based on a breach of the standard of care established by the statute, and awarded damages to the plaintiff. The defendant appealed, arguing again that he never had a duty in the first place.

What standard of review is the appellate court likely to apply in ruling on the defendant’s appeal?

AHarmless error review.

BThe appellate court will only rule for the defendant-appellant if it finds that there was a complete lack of proof of the defendant’s negligence, such that no reasonable jury could have found the defendant liable.

CThe appellate court will only rule for the defendant-appellant if it finds that the district court judge abused her discretion in denying the defendant’s motion to dismiss.

DDe novo review.

A
112
Q

At the start of the trial of a defendant and a codefendant for robbery, the codefendant and her attorney offered to give the prosecutor information about facts that would strengthen the prosecutor’s case against the defendant in exchange for leniency toward the codefendant. The prosecutor refused the offer. Shortly thereafter, the codefendant committed suicide. During the defendant’s trial, the prosecutor called the codefendant’s attorney and asked him to relate the information that the codefendant had revealed to the attorney.

Is the attorney’s testimony admissible?

AYes, because the codefendant intended to disclose the information.

BNo, because the plea discussion was initiated by the codefendant rather than by the prosecutor.

CNo, because the codefendant’s communications are protected by the attorney-client privilege.

DYes, because the information the codefendant gave to her attorney revealing her knowledge of the crime would be a statement against the codefendant’s penal interest.

A
113
Q

In a diversity case, a plaintiff sued a defendant for fraud and false advertising. The defendant counterclaimed for breach of contract. The jury returned a verdict finding both parties liable on all claims. The jury awarded the plaintiff $1 in damages and the defendant $150,000 in damages. Governing law permitted damages as high as $200,000 for the plaintiff, which the plaintiff had demanded in all papers submitted to the court. In contrast, the contract that formed the basis for the defendant’s counterclaim had a liquidated-damages clause that provided damages of $50,000 for any breach. The defendant had requested $50,000 in damages in all papers submitted to the court. Both parties used post-verdict motions to challenge the amount of damages, seeking judgment as a matter of law for the damages amounts they had requested or, in the alternative, new trials. Neither party challenged the jury’s determinations of liability.

What action, if any, may the federal district court take to correct the jury’s damages awards?

AThe federal district court may increase damages for the plaintiff up to $200,000 and decrease damages for the defendant to $50,000 with the defendant’s consent.

BThe federal district court may increase damages for the plaintiff up to $200,000 and grant the defendant a new trial on the breach-of-contract claim.

CThe federal district court may award the plaintiff a new trial on the fraud and false-advertising claims and decrease the defendant’s damages to $50,000 with the defendant’s consent.

DThe federal district court may not alter the jury’s damages awards for either party and may only award the parties new trials if the jury’s damages awards are against the clear weight of the evidence.

A
114
Q

Which of the following tools, if any, may the plaintiff use with a subpoena to obtain discovery from the witness without seeking leave from the federal district court?

-pretrial justice-

A
  • a party may serve an unlimited number of requests for production of documents and things, on both parties and nonparties alike
  • Without leave from the court, each side in a litigation in federal district court may take no more than 10 depositions.
  • A party may serve a maximum of 25 interrogatories, including distinct subparts, on another party. While a party may serve more than 25 interrogatories with leave from the court, interrogatories may not be used to seek discovery from persons and organizations that are not parties to the litigation.
115
Q

A state court found that the U.S. Army had violated the state’s Air Quality Act by failing to follow the proper procedures in removing asbestos-containing pipes at an Army base located in the state and imposed a civil penalty on the Army. The United States did not dispute that the Army violated the state law by failing to comply with the state’s asbestos-handling regulations; however, the United States appealed, alleging that the penalty’s imposition violated the federal government’s sovereign immunity.

Are there any circumstances in which a court would uphold the penalty?

A
  • Yes, if Congress expressly waived immunity in this context.
  • Broadly speaking, the federal government is immune from direct state regulation.
  • However, Congress may waive this immunity and require the federal government to comply with state law by clearly indicating federal consent to be bound.
  • Therefore, a court would uphold the penalty if Congress had explicitly required that the Army comply with state asbestos regulations.
116
Q

A state statute stated that “all students enrolled in public schools must present proof that they are currently vaccinated in accordance with the U.S. Centers for Disease Control and Prevention’s current recommendations.” A student challenged the statute as an unconstitutional interference with the free exercise of the student’s religion.

What standard of review, if any, applies to the student’s claim?

A
  • Minimal scrutiny
  • Not, Strict Scrutiny
  • if a generally applicable state law places only an incidental burden on religion, it is evaluated under minimal scrutiny, which presumes the law is valid and requires the challenger to show that the law is not rationally related to any legitimate government interest
  • Although some laws target or penalize religion, other laws place only incidental burdens on religion.
  • These laws are reviewed differently from laws that deliberately target or penalize religion.
117
Q

The mailbox rule, in which an offeree’s acceptance is effective upon proper dispatch, does not apply to option contracts. An offeree’s acceptance of an offer being kept open by an option contract is effective when the acceptance is received, not when it is properly dispatched.

A

A retiring store owner offered to sell his business to a prospective buyer. The buyer wanted time to think the offer over, so the store owner agreed to keep the offer open for 10 calendar days in exchange for $500. The option contract stated that the store owner would revoke the offer if the prospective buyer did not accept by the close of the tenth day. On day 5, the prospective buyer decided to purchase the business and mailed an acceptance to the store owner. While the acceptance was properly addressed, the store owner did not receive it until day 11, due to several mail holidays in the interim. By that time, the store owner had initiated negotiations with a second prospective buyer.

Was the first prospective buyer’s acceptance effective?

No, because the first prospective buyer’s acceptance was not effective until it was received on day 11, after the option expired.

118
Q

The Sixth Amendment right to counsel attaches to critical stages of pretrial proceedings.

  • a suspect has a right to counsel during in-person identifications once adversarial proceedings have begun, even pre-indictment.
  • However, a post-indictment photo array, or show-up, at which the suspect is not physically present, is not a critical stage of pretrial proceedings; therefore, a suspect does not have a right to the assistance of counsel.
A

A suspect was charged with multiple burglaries in a single town. Because there were many people claiming to have witnessed aspects of the burglaries, police sought to have the witnesses identify the suspect. Police conducted lineups, in which witnesses were asked to identify the suspect from among a group of persons of similar appearance. Police also conducted show-ups, in which witnesses were asked to identify the suspect from a photo array that included the suspect’s photo and photos of other persons of similar appearance.

Was the suspect entitled to the assistance of counsel in any of the witness identification proceedings?

Yes, suspects are entitled to assistance of counsel for pretrial lineups because they are a critical stage of pretrial proceedings.

not - No, suspects are not entitled to assistance of counsel for pretrial lineups or show-ups because they are not critical stages of pretrial proceedings.

119
Q

A contract is voidable if a party’s assent is induced by an untrue assertion that is fraudulent or material, provided that the party’s reliance on the untrue assertion is justified.

An untrue assertion must be fraudulent or material in order for a contract to be voidable for fraud or misrepresentation. In contrast, whether a misstatement goes to a basic assumption underlying the contract is relevant to whether the contract is voidable for mistake, not for fraud or misrepresentation.

A

A prospective buyer was interested in purchasing a car-wash business that was for sale. The buyer asked the current business owner to provide revenue, profit, and loss statements for the business going back five years. The business owner took an aggressive accounting approach that artificially inflated the car wash’s profits by 30 percent; the owner worried that otherwise, no one would purchase the car wash. Had the prospective buyer researched the issue independently by watching traffic enter the car wash, the buyer would have discovered that the profit statements were inflated. However, the prospective buyer conducted no independent investigation and trusted the statements the car-wash owner provided. In light of the rosy picture painted in the statements, the prospective buyer purchased the car wash. After making the first of three installment payments to the business owner, the buyer learned that the profit statements had been overstated.

May the buyer avoid the sales contract due to misrepresentation?

  • Yes, because the car-wash owner’s misrepresentation went to a basic assumption underlying the sales contract.
  • Yes, because the inflated profit statements were likely to induce the prospective buyer’s assent.
  • No, because the prospective buyer could have discovered that the profit statements were inflated with the exercise of reasonable care.
  • No, because the sale was an arm’s-length transaction with no special relationship of trust or confidence between the car-wash owner and the buyer.
120
Q

The Double Jeopardy Clause bars the government from appealing a fact finder’s verdict of acquittal. Similarly, the government may not appeal a trial court’s dismissal of a charge based on insufficient evidence or judgment of acquittal. However, the government may appeal the pretrial dismissal of charges, because double jeopardy has not yet attached. Therefore, the government may appeal only the pretrial dismissal of the burglary count.

A

A defendant was charged with one count of burglary and one count of aggravated assault. The trial court dismissed the burglary count prior to trial, and the jury acquitted the defendant of the aggravated assault count.

What, if anything, may the government appeal?

  • The government may appeal the acquittal of the aggravated assault count.
  • The government may appeal the pretrial dismissal of the burglary count
  • The government may appeal both the acquittal of the aggravated assault count and the pretrial dismissal of the burglary count.
  • The government may appeal neither the acquittal of the aggravated assault count nor the pretrial dismissal of the burglary count.
121
Q

Rule 612 of the Federal Rules of Evidence, if a writing or tangible thing is used to refresh a witness’s recollection, it is usually not permitted to be admitted into evidence.

neither exempt from the definition of hearsay nor does it qualify as an exception to the rule against hearsay.

A

A witness to a hit-and-run automobile accident jotted the automobile’s license plate number on the cover of a magazine she had in her car. At the resulting personal-injury trial, the plaintiff called the witness to testify about what she had seen. During her testimony, the plaintiff was unable to remember the license plate number of the automobile involved in the accident.

Under which of the following circumstances, if any, may the court permit the plaintiff to introduce the magazine cover into evidence?

There are no circumstances that would permit the plaintiff to introduce the magazine cover into evidence.

122
Q

Under the rule of convenience, unless a grant shows a contrary intent, a class closes when the first member of the class becomes eligible to receive a distribution. At that point, only individuals who are actual class members at the time of closing may participate in the class. Individuals who are not yet eligible to join the class receive nothing

A

A woman left her estate upon her death to her eldest son for life, then to her grandchildren then living. Upon the woman’s death, she had one eldest son and three grandchildren. After the woman’s death, her third grandchild died, leaving behind a spouse and young child as heirs. The woman’s daughter also gave birth to a fourth grandchild. Shortly thereafter, the woman’s eldest son died. Several months after the eldest son died, the woman’s fifth grandchild was born to her daughter.

Who has possession of the woman’s estate?

The first, second, and fourth grandchildren.

the class gift to the woman’s grandchildren closes upon the death of the woman’s eldest son. The death of the woman’s eldest son ends the son’s life estate and is the point at which any grandchildren are eligible to receive a distribution of the estate

123
Q

Conspiracy is (1) an agreement (2) between two or more people (3) to commit a crime. Unlike attempt, the crime of conspiracy does not merge with the completed target offense. At the heart of conspiracy is the existence of an agreement, explicit or implicit, for the joint purpose of committing the underlying crime. At common law, a conspiracy had to be at least bilateral, meaning that two or more people had to agree genuinely and sincerely to commit the target crime. Therefore, if the agreement were between the defendant and someone merely feigning agreement, such as an undercover police officer, the defendant could not be convicted of conspiracy because the agreement was unilateral.

A

An undercover police officer, who was investigating interstate illegal drug trafficking, asked a suspected drug trafficker if she would transport illegal drugs from State A to State B for distribution. The suspected drug trafficker agreed. The pair arranged to meet the next day, when the undercover officer would deliver the drugs to the suspected drug trafficker. When the pair met, the undercover officer gave a suitcase containing illegal drugs to the suspected drug trafficker, who put the suitcase in the trunk of the car she was driving. Police officers, who had been surveilling the transaction, swarmed the scene, and arrested the suspected drug trafficker.

Which of the following facts, if true, would support acquitting the suspected drug trafficker of conspiracy to distribute illegal drugs under the common law?

The conspiracy was unilateral.

Not, The conspiracy did not merge with the target offense.

124
Q

If the seller is a merchant with respect to goods of the kind being sold, then the Uniform Commercial Code implies a warranty of merchantability, regardless of the language of the contract. In general, goods are merchantable if they are fit for the ordinary purpose for which goods of the kind are used. U.C.C. § 2-314 (2002). The implied warranty of merchantability may be disclaimed by language explicitly mentioning merchantability. A written disclaimer must be conspicuous, for example, by prominently setting apart disclaimer text, including the text in a location the buyer is likely to see, or using larger or bolder font than the surrounding text.

A

A tomato farmer contracted to sell tomatoes to a national grocery-store chain. The farmer included a clause in the contract that disclaimed any implied warranty of merchantability. The disclaimer clause was a standard, numbered paragraph in the warranties section of the contract document. The clause appeared in the same font style and size as the remainder of the contract. The farmer ultimately delivered a batch that had a significant number of unsellable tomatoes due to rot. The contract contained no express terms about the tomatoes’ required quality.

Has the farmer breached the contract?

Yes, because the farmer’s disclaimer of the implied warranty of merchantability was not conspicuous.

not: No, because the farmer expressly disclaimed the implied warranty of merchantability.

125
Q

Character evidence is evidence regarding a person’s mental and moral qualities. Character evidence is generally not admissible to show that a person acted in conformity with that character (i.e., propensity or proclivity evidence) because of the potential that a jury would make improper inferences from the evidence. However, it may be admissible for purposes other than showing a person acted in conformance with that character, for example, to impeach a witness or, in a criminal case, to prove motive or opportunity.

A

A defendant in a breach of contract action called his parish priest to testify that the defendant was a trustworthy person of integrity.

For which of the following reasons, if any, may this testimony be admitted?

None

126
Q

The concept of sovereign immunity provides, in part, that a state is immune from being sued by private parties without that state’s consent. In the U.S. Constitution, this immunity is reflected in the Eleventh Amendment, which provides that federal judicial power “shall not be construed to extend to any suit . . . commenced or prosecuted against [a state] by Citizens of another State.” The Eleventh Amendment also generally bars private-plaintiff suits against a state in the state’s own courts for violations of federal law. owever, the Eleventh Amendment does not bar suits against local governments such as counties and municipalities. That is, the protections of the Eleventh Amendment are limited to the states themselves and do not extend to lower levels of government. Therefore, the Eleventh Amendment does not bar the citizen’s action against his local municipality’s voting commission.

A

A private citizen filed suit in federal court, seeking money damages and injunctive relief, against his municipality’s voting commission. The citizen claimed that the commission had implemented voting procedures that unconstitutionally burdened the citizen’s right to vote.

Does the U.S. Constitution’s Eleventh Amendment bar this action?

  • No, because the Eleventh Amendment only bars suits seeking equitable relief.
  • No, because the Eleventh Amendment does not bar suits against local governments.
  • Yes, because the Eleventh Amendment bars suits alleging constitutional violations.
  • Yes, because the Eleventh Amendment ensures that all state and local governments have sovereign immunity.
127
Q

If a jury’s answers to written questions are inconsistent with one another, and at least one answer is inconsistent with the general verdict, then a federal district court must (1) direct the jury to reconsider or (2) order a new trial. Fed. R. Civ. P. 49(b)(4). Here, the jury has returned a general verdict for the plaintiff. However, the jury responded in one answer to a written question that the defendant did not breach the contract. Thus, at least one of the jury’s answers is inconsistent with the general verdict. Moreover, the jury’s answer that the defendant did not breach the contract is inconsistent with the jury’s other answer that the defendant did not deliver goods in mint condition, as the contract required. Because the jury’s answers to written questions are inconsistent with each other and at least one of the jury’s answers is also inconsistent with the jury’s general verdict, the federal district court here has two options. The court may urge the jury to reconsider, or it may order a new trial.

A

Following trial in federal district court of a plaintiff’s breach-of-contract claim in a diversity action, a jury returned a general verdict accompanied by answers to written questions. Prior to the jury’s deliberations, the court had instructed the jury that the contract between the parties required the defendant to deliver goods in mint condition by 2:00 p.m. on the agreed-upon date. The court further instructed that the parties had defined the defendant’s failure to deliver goods in the required condition and at the promised time as a breach of the contract. The jury returned a general verdict for the plaintiff. In responding to three written questions, the jury indicated that (1) the defendant did not deliver goods in mint condition, (2) the defendant did deliver the goods at 2:00 p.m. as promised, and (3) the defendant did not breach the contract.

What must the federal district court do with the jury’s general verdict and answers to written questions?

The court must direct the jury to reconsider its general verdict and answers, or the court must order a new trial.

128
Q

although police reports are records of matters observed by a public official while under a legal duty to report them, the rule expressly prohibits the introduction of police reports in a criminal case unless they are being offered against the government

A

In a criminal prosecution for fraud, the government sought to introduce the investigating officer’s reports under the public- records exception to the rule against hearsay.

Should the court admit the reports under the public-records exception to the rule against hearsay?

No, because the reports are not admissible in a criminal case.

129
Q

A private actor may be treated as a state actor in particular circumstances, such as if the private actor is performing a function that traditionally has been exclusively reserved to the government.

A

A mining company owned a small town in State A. A town resident was arrested for distributing religious materials, in violation of town policy. The resident challenged her arrest as a violation of her First Amendment rights.

Which of the following grounds would support a finding that the town violated the resident’s First Amendment rights?

The mining company was performing a function that traditionally was exclusively reserved to the government.

NOT - The mining company was formed under and subject to the laws of State A.

130
Q

A party who is entitled to a jury trial on an issue must serve a written demand on the other parties within 14 days after service of the last pleading on the issue. Fed. R. Civ. P. 38(b)(1). Failure to file a timely jury demand waives a right to a jury trial.

A

An author sued a publisher for copyright infringement in federal district court. The publisher timely answered the complaint 21 days later. The publisher’s answer included various defenses, including noninfringement, and also asserted a counterclaim for breach of contract, claiming that the author had failed to deliver a manuscript according to the schedule defined in the author’s contract. The author timely answered the counterclaim 10 days after the publisher filed its answer. The author’s answer to the counterclaim included several defenses to the breach-of-contract claim but did not otherwise address the copyright-infringement claim. Ten days after the author answered the counterclaim, the author filed and served on the publisher a demand for a jury trial on the author’s copyright-infringement claim.

Did the author waive his right to a jury trial on the copyright-infringement claim?

Yes, because more than 14 days passed between the publisher’s answer and the author’s demand.

131
Q
A